You are on page 1of 141

z

Section – 1 Lecture Notes -1 (A) REAL ANALYSIS

Algebraic properties of ℝ - On the set of real numbers there


are two binary operations denoted by + and ∙are called
addition and multiplication,respectively. These operations
satisfy the following properties.

1. a + b = b + a, ∀ a, b ∈ℝ(Commutativiti w.r.t addition)


2. (a + b) + c=a+(b + c) ∀ a ,b ,c∈ ℝ(Associativity of
addition)
3. There exists an element 0 in ℝ Such that 0+a = a = a+0
∀a∈ℝ (Existence of zero element)
4. ∀ a ∈ ℝ ∃ -a ∈ ℝ : a+(-a) = 0 and (-a) + a = 0
5. a ∙ b = b ∙ a ∀ a, b ∈ ℝ (commutative of multiplication)
6. (a ∙ b) ∙ c = a ∙ (b ∙ c) ∀ a, b, c ∈ ℝ(Associative of
multiplication)
7. (Existence of unit element) ∃ 0 ≠ 1 ∈ ℝ : a ∙ 1 =1 ∙ a =a
∀a∈ℝ
8. (Existence of reciprocals) ∀ 0 ≠ a∈ℝ ∃ ∈ ℝ: a ∙ =1= ∙ a

9. a∙ (b + c) = (a ∙ b)+( a ∙ c)

and (b + c) ∙a = (b ∙ a) + (c ∙ a) ∀ a, b, c ∈ℝ.( Distributive


property of multiplication over additon).

Theorem

1. If z and a are elements in ℝ With z + a = a, then z = 0


2. If u and b ≠ 0 are elements in ℝ With u ∙ b = b, then u = 1
3. If a ∈ ℝ , then a ∙ 0 = 0.

 Notes prepared by sanjeev kumar shukla


 9971245238 or 8750558444
z

Proof –

1. z = z + 0 (Existence of zero element)

= z + (a + (-a)) = (z + a) + (-a)

= a + (-a) = 0.

2. If u and b ≠ 0 are elements in ℝ and u ∙ b = b

To Show that - u = 1

Consider u = u ∙ 1 (Existence of unit)

= u (b ∙ ) (Existence of reciprocals)

= (u ∙ b) (using association of multiplication

= b∙ ( u∙b= bis guven)

⇒ u = b∙ = 1

⇒ u = 1

3. If a ∈ℝ , then a ∙ 0 = 0

a + a ∙ 0 = a ∙ 1 + a ∙ 0 (Existence of unit)

= a (1 + 0) = a ∙ 1 (Distributivity)

= a (Existence of unit)

⇒ a ∙ 0 = 0 (By (a))

Theorem
1. If 0 ≠ a ∈ ℝ are such that a ∙ b = 1 ⇒ b =

2. If a ∙ b = 0 then either a = 0 or b = 0

 Notes prepared by sanjeev kumar shukla


 9971245238 or 8750558444
z

Proof –

(1). b = b ∙ 1 (Existence of unit)

= b (a∙ ) (a≠0, Existence of reciprocal)

= (ba) ∙ (Associativity)

= (ab) ∙ (Commutativity of multiplication)

= (1 ∙ ) (ab = 1)

⇒ b = (Existence of unit element)

(2). let a ∙ b = 0, If a = 0 we are done , if a ≠ 0 ,1/a exists.

Now , b = b ∙ 1 = b ) (Existence of reciprocals)

= b(a ∙ ) = (ba) = (ab) =0 ∙ = 0

⇒ b = 0

Notation –

1. a + (-b) = a – b ∀ a, b ∈ ℝ

2. a ∙ ( ) = if b ≠ 0

3. a(n+1) = (an)a n ∈ ℕ

4. a1 = a

5. If a ≠ 0 then a0 = 1 and a-1 = ,Also if n ∈ ℕ , we will

write a(-n) = ( )n

 Notes prepared by sanjeev kumar shukla


 9971245238 or 8750558444
z

Rational number –

Any number that can be written as where a, b ∈ ℤ and b ≠ 0,

is called a rational number. The set of all rational numbers is


denoted by ℚ.

Irrational number –

Any number which is not rational is called irrational.

OR

A number X is called irrational iff ∄ a and b(≠0) in ℤ such that

X =

Theorem –There does not exists a rational number


‘r’such that r2 = 2.

Proof– Let if possible ∃ rational number

r = (q ≠ 0, p, q ∈ ℤ) :

( ) = 2.

Note - without the loss of generality we can assume that

there is no common Factor b/w p and q, and p and q both are

positive ,Now ( ) =2 ⇒ p2 = 2q ⇒ p2 is even

(p is odd ⇒ p = 2n-1 , n ∈ ℤ

Then p2 = (2n-1)2

= 2(2n2-2n+1)-1, is odd)

 Notes prepared by sanjeev kumar shukla


 9971245238 or 8750558444
z

Also p and q do not have 2 as a common factor

∴ q is odd …….. (1)

Now as p is even p = 2m

Where m ∈ ℕ :

⇒ 4m2 = 2q2 ⇒ q2 = 2m2

⇒ q2 is even ⇒ q is even ……… (2)

From (1) and (2) we have a contradiction.

And hence there is no p and q in ℤ such that

( )2 = 2 ⇒ ∄ any r ∈ ℚ : r2 = 2.

ORDER PROPERTY OF ℝ -

There is a non empty subset P of ℝ, called

The set of positive real numbers ,

That satisfy the following properties.

1. If a, b belong to P , then a + b ∈ P.
2. If a, b ∈ P then a ∙ b ∈ P
3. If a ∈ℝ then exactly one of the following is true..
a ∈ P , a= 0 , -a ∈ P.

3 r d property is called trichotomy property


Negative Set –

Define a set {-a : a ∈ P} , then this set is called set of negative


numbers.

 Notes prepared by sanjeev kumar shukla


 9971245238 or 8750558444
z

Notations –

1. If a ∈ P we write , a > 0
2. If a ∈ P U {0} , we write a ≥ 0
3. If -a ∈ P we write a < 0
4. If -a ∈ P U {0} , we write a ≤ 0

a > 0 , ⇒ a is positive real number

a ≥ 0 , ⇒ a is non-negative real number

a < 0 , ⇒ a is negative real number

a ≤ 0 , ⇒ a is non-positive real number

Definition - Let a , b be the elements in ℝ

(a). If a-b ∈ P, we write a > b or b < a

(b). If a-b ∈ PU{0}, we write a ≥ b or b ≤ a

Theorem - Let a, b, c be any elements of ℝ

(a). If a > b and b > c , then a > c

(b). If a > b then a + c > b +c

(c). If a > b and c > 0 , then ca > cb

(d). If a > b and c < 0, then ca < cb

Note -(b)is called monotonic law of addition.

(c)is called monotonic law of multiplication.

 Notes prepared by sanjeev kumar shukla


 9971245238 or 8750558444
z

Proof -

(a). Given - a > b and b > c

⇒ a – b ∈ P and b – c ∈ P

To Show that - a > c i.e a – c ∈ P

As a – b ∈ P and b – c ∈ P

( By property 1.of P ) we have (a - b) + (b - c) ∈ P

⇒ a + (-b + b) - c ∈ P

⇒ a - c ∈ P ⇒ a > c.

(b). If a > b, then a + c > b + c

Proof- Given - a > b ⇒ a - b ∈ P

Now (a + c) - (b + c) = a + c - b - c

= a + c - c - b

= a - b ∈ P

⇒ (a + c) - (b + c) ∈ P ⇒ a + c > b + c.

(c). Given - a > b ⇒ a - b ∈ P

and c > 0 ⇒ c ∈ P , ∴ by property 2nd of P

c(a - b) ∈ P ⇒ ca - cb ∈ P

⇒ ca > cb.

(d). Given - a > b ⇒ a - b ∈ P

 Notes prepared by sanjeev kumar shukla


 9971245238 or 8750558444
z

And c < 0 ⇒ -c ∈ P

∴ again by property (2) of +ve set P

(-c)(a - b) ∈ P

⇒ -ca + cb ∈ P

⇒ cb - ca ∈ P

⇒ (cb > ca) .

Theorem -

(a) If a ∈ ℝ and a ≠ 0 ⇒ a2 > 0

(b) 1 > 0

(c). If n ∈ ℕ

Proof -

(a). Given - a ∈ ℝ and a ≠ 0

To show - a2 > 0

As a ∈ ℝ and a ≠ 0

⇒ a ∈ P or –a ∈ P

Now if a ∈ P then a2 = a ∙ a ∈ P

⇒ a2 ∈ P or a2 > 0

Now if -a ∈ P

 Notes prepared by sanjeev kumar shukla


 9971245238 or 8750558444
z

a2 = (-a)(-a) ∈ P ( -a ∈ P ⇒ (-a)(-a) ∈ P)

⇒ a2 ∈ P ⇒ a2 > 0

(b). Now as 0 ≠ 1 ∈ ℝ

∴ by (a) 12> 0

⇒ 1 = 12 > 0 ⇒ 1 > 0

(c). We’ll show using mathematical induction –

For n = 1 ; 1 > 0

Now assume result is true for n = k ⇒ k ∈ P

Now we’ll show that result is also true for n = k +1

Now k ∈ P and 1 ∈ P

⇒ k +1 ∈ P (using 1st property of P)

⇒ result is true for n = k +1 ∴ result is true for all


n ∈ ℕ⇒ n > 0 ∀ n ∈ℕ .

Absolute value –

Let a be any real number then absolute value of a is


denoted by |a| and is defined as.

|a| = {

 Notes prepared by sanjeev kumar shukla


 9971245238 or 8750558444
z

Theorem –

(a). |ab| = |a| |b| ∀ a, b ∈ ℝ

(b). |a|2 = a2 ∀ a ∈ ℝ

(c). if c ≥ 0, then |a| ≤ c iff -c ≤ a ≤ c

(d). -|a| ≤ a ≤ |a| ∀ a ∈ ℝ

Proof-

(a). if a = 0 or b = 0 ⇒ |ab| = |a| ∙ |b|

So assume a ≠ 0 , b ≠ 0 .

Case – 1 a > 0, b > 0 ⇒ ab > 0 ⇒ |ab| = ab

Now |ab| = ab = |a|∙ |b|

(∵ a > 0 ⇒ |a| = a & b > 0 ⇒ |b| = b )

Case – 2 a < 0, b < 0 ⇒ ab > 0 ⇒ |ab| = ab

Now |ab| = ab = (-a)(-b) = |a|∙


|b| (∵ b < 0 ⇒ |b| = -b, a < 0 ⇒ |a| = -a )

Case – 3 a < 0, b > 0 ⇒ ab < 0 ⇒ |ab| = -ab

⇒ |ab| = (-a)b = |a||b|

( ∵ a < 0 ⇒ |a| = -a, b > 0 ⇒ |b| = b )

Case – 4 a > 0, b < 0 ⇒ ab < 0 ⇒ |ab| = -ab

⇒ |ab| = a(-b) = |a||b|

( ∵ a > 0 ⇒ |a| = a, b < 0 ⇒ |b| = -b )

∴ in possible cases we have

|ab| = |a||b|
 Notes prepared by sanjeev kumar shukla
 9971245238 or 8750558444
z

(b). ∀ a ∈ ℝ : a2 ≥ 0

⇒ a2 = |a|2 = |a ∙ a| = |a| ∙ |a| = | a|2

⇒ |a|2 = a2

(c). c ≥ 0, then |a| ≤ c iff -c ≤ a ≤ c

Note - To prove this we’ll prove the following

Max {a, -a} = |a|

When a = 0 the result is true assume a ≠ 0.

Case – 1 a > 0 ⇒ |a| = a …….. (1)

a > 0 ⇒ -a < 0 ⇒ a > -a ⇒ max {a, -a} = a

⇒ max {a, -a} = a = |a| (By (1) )

⇒ max {a, -a} = |a|

Case – 2 a < 0 ⇒ |a| > -a ……… (2)

a < 0 ⇒ -a > 0 ⇒ -a > a ⇒ max {a, -a} = -a

⇒ max {a, -a} = -a = |a| (By (2) )

⇒ max {a, -a} = |a|

Now we’ll show that |a| ≤ c ⟺ -c ≤ a ≤ c

|a| ≤ c ⟺ max {a, -a} ≤ c

⟺ a ≤ c & -a ≤ c

⟺ a ≤ c & a ≥ -c

⟺ -c ≤ a ≤ c

 Notes prepared by sanjeev kumar shukla


 9971245238 or 8750558444
z

(d). Now |a| ≥ 0

Do yourself -

Triangular Inequality –

If a, b ∈ ℝ then |a + b| ≤ |a| + |b|

Proof -

Case - 1 a + b > 0

|a + b| = a + b ….. (1)

and we know that a ≤ |a|, b ≤ |b|

⇒ a + b ≤ |a| + |b| ….. (2)

Using (2) in (1) we get |a + b| ≤ |a| + |b|.

Case - 2 a + b < 0

|a + b| = -(a + b)

= -(a) + -(b) ……. (3)

Now we know that

-(a) ≤ |a|, -(b) ≤ |b|

⇒ -(a) + -(b) ≤ |a| + |b| …… (4)

Using (4) in (3) we get |a + b| ≤ |a| + |b|.

Case - 3 a + b = 0 ⇒ a = -b and |a + b| = 0

⇒ |a| = |-b| = |b|

⇒ |a| + |b| = 2|a|

 Notes prepared by sanjeev kumar shukla


 9971245238 or 8750558444
z

⇒ |a + b| = 0 ≤ |a| + |b|

⇒ |a + b| ≤ |a| + |b|

Corollory - If a, b ∈ ℝ then

(a). | |a| - |b| | ≤ |a - b|,

(b). |a - b| ≤ |a| + |b|.

Proof - |a| = |a - b + b|

≤ |a - b| + |b| (using triangle inequality)

⇒ |a| - |b| ≤ |a - b| ….. (1)

Now |b| = |b – a + a| ≤ |b - a| + |a|

⇒ |b| - |a| ≤ |b - a| = |a - b| ….. (2)

⇒ max {|a| - |b|, |b| - |a| } ≤ |a - b|

⇒ max {|a| - |b|, -(|a| - |b|) } ≤ |a - b|

⇒ | |a| - |b| | ≤ |a - b|.

Definition- Let a ∈ ℝ and ε> 0. Then the

ε–neighborhood of a is the set vε(a) defined as


vε(a) = { x ∈ ℝ : |x – a| <ε}.

 Notes prepared by sanjeev kumar shukla


 9971245238 or 8750558444
z

Thoerem - Let a ∈ ℝ . If x belongs to the neighborhood


vε(a) ∀ ε > 0, then x = a.

Proof - Since 0 ≤ |x – a| < ε ∀ ε > 0

∴ |x – a| = 0 (∵ if 0 ≤ a < ε ∀ ε > 0 then a = 0 )

⇒ x = a.

Q1: If a, b ∈ ℝ and b ≠ 0 show that

(a). |a| = √ (b). | ⁄ | = ⁄

Sol: (a). Case – 1 a > 0 ⇒ |a| = a

⇒ |a|2 = a2

⇒ |a| = √

Case – 2 a < 0 ⇒ |a| = -a

⇒ |a|2 = -(a)2 = a2

⇒ |a| = √

Case -3 a = 0 ⇒ |a| = 0

⇒ |a|2 = 02

⇒ |a| = √ = √

(b). | ⁄ | = ⁄

 Notes prepared by sanjeev kumar shukla


 9971245238 or 8750558444
z

Case -1 a > 0, b > 0 ⇒ > 0 ⇒ | | = =

(∵ a = |a|, b = |b| )

Case -2 a < 0, b < 0 ⇒ > 0 ⇒ | | = = ⁄ =

(∵ |a| = -a, |b| = -b| , a <0, b < 0)

Case -3 a < 0, b > 0 ⇒ < 0 ⇒ | |= ( )= ⁄ =

(∵ |a| = -a, |b| =b , a <0, b> 0)

Case -4 a > 0, b > 0. Do yourself.

ε–neighborhood

Let a be any real number and let ε > 0 be any number then

ε – neighborhood of “a” is define d as the set

vε(a) = { x ∈ ℝ : |x – a| < ε} = (a – ε, a + ε).

Neighborhood

Let a be any real number thena non – empty set V is called


neighborhood of “a” iff ∃ ε > 0 :

vε(a) ⊆ V i .e (a – ε, a + ε) ⊆ V.

Note - Every ε – neighborhood of “a” is neighborhood of “a”

Let V = vε(a)

We’ll show that V is a nbhd of a.

As vε(a) is an ε – nbhd of ‘a’ also vε(a) ⊆ vε(a) = V

⇒ vε(a) ⊆ V ⇒ V is a nbhd of a

 Notes prepared by sanjeev kumar shukla


 9971245238 or 8750558444
z

Open Set - A set G is said to be an open set if for each x ∈ G

∃ a nbhd V of x such that V ⊆ G.

Closed Set – A set F is said to be closet set if it’s complement

Fc = ℝ - F is an open set.

Theorem -

Let A be subset of ℝ then A is open iff ∀ x ∈ A ∃ εx > 0 :

]X - εx , X+ εx [ ⊆ A

Proof - First assume A is open then ∀ x ∈ A ∃ nbhd of V


of x : V ⊆ A ……..(1)

Now V is nbhd of x ∴ ∃ ε x > 0 : ] X - εx , X+ εx [ ⊆ V ……… (2)

From (1) & (2) we have ] X - εx , X+ εx [ ⊆ A

Conversely – Assume that ∀ x ϵ A ∃ εx > 0 : ] X - εx , X+ εx [ ⊆ A

To show that - A is open I .e To show that ∈ nbhd of V of x


∈ x ∈ A

Let x ∈ A (be any element)

By our assumption ∃ ε x > 0 : ] X - εx , X+ εx [ ⊆ A

Let V = ] X - εx , X+ εx [ ⊆ A then V is a nbhd of x

⇒ V ⊆ A ∴ A is an open set .

 Notes prepared by sanjeev kumar shukla


 9971245238 or 8750558444
z

Note - From now onwards whenever we need to show A is


open we’ll show that

∀ x ϵ A ∃ εx > 0 : ] X - εx , X+ εx [ ⊆ A .

Remark - G is open iff ∀ x ∈ G ∃ ε > 0 :

vε(x) ⊆ G ⇒ G is neighborhood of x

⇒ G is open iff it is neighborhood of each of it’s points.

Example Every open interval of the type ]a, b[ is an open set.

Proof - Consider ]a, b[ Let x ∈ ]a, b[ be any point, we shall


show ]a, b[ is nbhd of x.

Let εx = min {x – a, b - x} then ]x - εx, x + εx [ ⊆ ]a, b[

(∵ εx = min {x – a, b - x} ⇒ εx ≤ x – a, εx ≤ b - x )

⇒ a ≤ x - εx & x + εx ≤ b )

⇒ ]x - εx , x + εx [ ⊆ ]a, b[

And hence ]a, b [ is open set .

Example – ℝ, The Set of reals is an open set .

Proof - Let x ∈ ℝ , then ∀ℰ> 0

]x - εx , x + εx [ ⊆ ℝ⇒ℝ is open set.

Example – Set of rationals Q is not open set .

Proof -Let x ∈ Q,then ∀ x ∈ Q and ∀ ℰ> 0 ]x - εx , x + εx [ ⊈ Q


(∵ ]x - εx , x + εx [ contains infinite no. of rationals which can
be in Q) ⇒ Q can’t be open.

 Notes prepared by sanjeev kumar shukla


 9971245238 or 8750558444
z

Question - Empty set is an open set.

Solution - Since ∅ is a set with no elements in it ∴ there


is no element in ∅ of which it is not a nbhd

⇒ ∅ is nbhd of each of its points

⇒ ∅ is open set.

Question – Any finite set is not open.

Solution - Let S be any finite set we’ll show S is not nbhd of


any of its points. Let x ∈ S be any

Then ∀ ε > 0

]x - εx , x + εx [ ⊈ S (∵ ]x - εx , x + εx [ is infinite
set while s is finite set)

⇒ S is not open.

Question – Every open interval of the type ] -∞, a[ is open.

Proof

- Let x ∈ ]-∞,a[ be any number ⇒ x + Ԑ < a

⇒ -∞ < x + Ԑ < a ⇒ -∞ < x - Ԑ < x + Ԑ < a

⇒ ]x - Ԑ, x +Ԑ [ ⊆ ]-∞,a[

⇒ ]-∞, a[ is open.

 Notes prepared by sanjeev kumar shukla


 9971245238 or 8750558444
z

Open Set Properties -

1.The union of an arbitrary collection of open subsets in ℝ is


open.

2.Intersection of finite collection of open sets is an open set.

Proof –

(a) Let {Gλ : λ ∈ Δ} be an arbitrary family of open set.

Let G = ⋃ we have to prove that G is open set .

Let x ∈ G (be any element)

⇒ x ∈ G = ⋃

⇒ x ∈ Gλ for λ ∈ Δ

As each Gλ is open set ∴ ∃ εx > 0 :

]x - εx , x + εx [ ⊆ Gλ ⊆ ⋃

⇒ ]x - εx , x + εx [⊆⋃ = G ⇒ G is open set .

(b). Let G1, G2 , G3 ……. Gn be open sets

To show that ⋂ is an open set.

Let x ∈ ⋂ (be any element)

⇒ x ∈ Gi ∀ i = 1, 2, 3….. n

As each Gi is an open set ⇒ ∃ εi > 0 , i = 1, 2, 3 …. n :

]x - εi, x+εi [ ⊆ Gi ∀ i = 1, 2, 3 ….n

Let Ԑ = min { Ԑ1, Ԑ2,……. Ԑn}

Then ]x - Ԑ, x + Ԑ [ ⊆ ] x - εi, x+εi [ ⊆ Gi ∀ i = 1, 2, 3 ….n


 Notes prepared by sanjeev kumar shukla
 9971245238 or 8750558444
z

⇒ ]x - Ԑ, x + Ԑ [ ⊆ Gi ∀ i = 1, 2, 3 ….n

⇒ ]x - Ԑ, x + Ԑ [ ⊆ ⋂

⇒ ⋂ is an open set.

Definition (Closed set) -

A set G is closed iff it’s complement is open.

Properties of Closed sets –

(a). Arbitrary intersection of closed sets is a closed set.

(b). Finite union of closed sets is a closed set.

Proof - Let {Gλ}λ∈Δ be a family of closed sets.

To show that – ⋂ is closed set.

i.e To show that (⋂ )cis open set

we know ⋂ ) = ⋂ ) ……. (1)

Now as Gλ is closed ∀ λ ∈ Δ (Gλ)c is open ∀ λ ∈ Δ

⇒ ⋃ ) is open set (∵ Arbitrary union of open


sets is an open set).

⇒ By (1) ⋂ ) is open

⇒ ⋂ closed set.

(b). Finite union of close d sets is a closed set.

Let G1, G2, …….. Gn be finite number of closed sets.

Then we have to show that ⋃ is closed i.e To


show that ⋃ ) is an open set.

 Notes prepared by sanjeev kumar shukla


 9971245238 or 8750558444
z

We know that ⋃ ) = ⋂ ) …… (1)

Now since Gi is closed for i = 1, 2, …. n

⇒ (Gi)c is open for i = 1, 2, …. N

⇒ ⋂ ) is open (∵ finite intersection of open sets is


open)

Note -1 Arbitrary intersection of open sets may not be open.

Example - Let F = { (- ) | n ∈ ℕ } a family of open sets.

We’ll show that ⋀ ℕ ( ) is not an open set

As ⋀ ℕ ( ) = {0} and {0} is not an open set,

being a finite set.

Note -2 Arbitrary union of closed sets may not be closed.

Example – Let F = {[ ] | n ∈ ℕ } then each member of F is

closed set (being closed intervals)

But ⋃ ℕ* + = (0, 2] is not closed.

Theorem – Every closed interval of the type [a, b] is a closed set.

Proof - Let F = [a, b] Then Fc = [a, b]c = (-∞,a)U(b, ∞) which


is open set (being union of open sets)

⇒ F is closed ⇒ [a, b] is closed set.

 Notes prepared by sanjeev kumar shukla


 9971245238 or 8750558444
z

Cluster point - (limit point)

A real number “x” is said to be limit point or cluster point of a set A


⊆ ℝ iff every Ԑ - nbhd of x contains a member of A other than ‘x’.
i.e ∀ Ԑ > 0

VԐ (x) ⋂ (A – {x}) ≠ ϕ

]x - Ԑ, x + Ԑ[ ⋂ (A – {x}) ≠ ϕ ∀ Ԑ > 0

Example – Consider the set S = ]a, b[ then ∀ Ԑ > 0 ]a-Ԑ,a+Ԑ[


contains infinitely many points of ]a,b[

⇒ a is limit point of ]a,b[

Similarly ∀ Ԑ > 0 ]b-Ԑ,b +Ԑ[ contains infinitely many points


of ]a,b[ ⇒ b is limit point of ]a, b[

Theorem - A set A is closed iff is contains all of it’s limit points.

Proof -First Let us assume that A is closed set .

Let x be any limit point of A

To show that x ∈ A.

Let x ∉ A ⇒ x ∈ ℝ ~ A

Now as A is closed ∴ ℝ ~ A is open.

⇒ ℝ ~ A is neighborhood of x

⇒ ∃ Ԑ > 0 : ]x - Ԑ, x + Ԑ[ ⊆ ℝ ~ A

 Notes prepared by sanjeev kumar shukla


 9971245238 or 8750558444
z

⇒ ]x - Ԑ, x + Ԑ[ ⋂ A = ∅

⇒ x is not a limit point of A which is a contradiction as x


is limit point of A

⇒ x ∉ A is not possible ⇒ x ∈ A

⇒ A contains all it’s limit points.

Conversely assume that A contains all it’s l imit points.

To show that – A is closed for that we need to show that


ℝ ~ A is open.

Let x ∈ ℝ ~ A (be any point)

⇒ x ∉ A ⇒ x is not limit point of A

⇒ ∃ Ԑ > 0 : VԐ(x) ⋂ A = ∅

⇒ VԐ(x) ⊆ ℝ ~ A

⇒ ℝ ~ A is nbhd of x

Since x was any arbitrary point of ℝ ~ A this


follows that ℝ ~ A is nbhd of each of it ’s points

⇒ ℝ ~ A is open

⇒ A is closed.

 Notes prepared by sanjeev kumar shukla


 9971245238 or 8750558444
z

Examples on Cluster point.

(1) Every Real number is cluster point of Q.

Solution – Let x ∈ ℝ be any real number then ∀ Ԑ > 0

]x - Ԑ, x + Ԑ[ contains infinitely many real numbers


which are rationals.

⇒ ]x - Ԑ, x + Ԑ[ ⋂ (Q – {x}) ≠ ∅

⇒ x is limit point of Q.

Definition – (Derived Set)

Let S be any subset of real numbers then derived set of S is


denoted by Sl and is defined as

Sl = {x : x is limit point of S}

Note – in view of above result and definition we can say


that Ql = ℝ

(2) Set of integers has no limit point.

Solution – Let ℤ denote the set of integers, we’ll show no real


number “x” can be limit point of ℤ

As x ∈ ℝ∴ we have two cases.

(1) x ∈ ℤ (2) x ∉ℤ

Case 1 x ∈ ℤ then ]x - [ is a neighborhood of

which contains no integers

 Notes prepared by sanjeev kumar shukla


 9971245238 or 8750558444
z

Other than x (between any two integers there


is distance of at least one unit)

⇒ x is not limit point of ℤ

Case 2 x ∉ ℤ then we can choose two integers n and


n+1 such that n< x < n+1

Then if we choose Ԑ = min {x-n, n+1-x}

]x-Ԑ, x+Ԑ[ will contain no integer

⇒ x can’t be limit point of ℤ

And hence no real number can be limit point of


⇒ ℤl = ∅

Example : ( ]a, b[ )l = [a, b]

Solution:

if x ∈ [a, b] then for every Ԑ > 0 , ] x-Ԑ, x+Ԑ [


contain infinitely many members of ]a,
b[

And ∴ one member of ]a, b [, other than ‘x’

∴ x is a limit point of ]a,b[

Now let x ∉ [a, b]

Case 1 x < a then choose (0<Ԑ<a-x)

 Notes prepared by sanjeev kumar shukla


 9971245238 or 8750558444
z

⇒ x + Ԑ<a and ∴ ]x-Ԑ, x+Ԑ[ does not contain


any member of [a, b]

⇒ x is not a cluster point of ]a, b[

Case 2 x > b, then choose 0<Ԑ<x-b

⇒ x-Ԑ > b

⇒ ∴ ]x-Ԑ , x+Ԑ[ does not contain any member


of ]a, b[

⇒ x is not a limit point of ]a, b[

Hence ( ]a, b[ )l = [a, b]

Example Set of integers is closed.

Solution: As ℤl = ∅ and ∅ ⊆ Z ⇒ ℤl ⊆ Z

⇒ ℤ contains all it’s limit points

⇒ ℤ is closed

Example Set of natural numbers has no limit point.

OR

(ℕ)l = ∅

And (ℕ)l = ∅ ⊆ ℕ⇒ℕ is closed

Example: Let S = { ℕ}

Then Sl = {0}

 Notes prepared by sanjeev kumar shukla


 9971245238 or 8750558444
z

Solution: Let Ԑ > 0 be any real number then by archimedian


property ∃n∈ℕ :

n > or n <

Now S and ∈ ]0-Ԑ, 0+Ԑ[

⇒ ∀ Ԑ > 0 ]0-Ԑ, 0+Ԑ[ contains a member of S ⇒


0 is cluster point of S

To show - No real number other than 0 can be limit


point of S

Let x ≠ 0 be any real number

Case 1 x < 0

Then choose 0<Ԑ<-x ; x+Ԑ < 0

∴ ]x-Ԑ, x+Ԑ[ does not contain a member of S

⇒ x can’t be limit point of S

Case 2 x > 1

Then choose 0<Ԑ<x-1

∴ ]x-Ԑ, x+Ԑ[ doest not contain any member of


S (∵ x-Ԑ >1 ⇒ ]x-Ԑ, x+Ԑ[ ⊆ ]1, ∞[)

Case 3 x = 1, the for Ԑ = 1/3 (x-Ԑ, x+Ԑ) = (2/3,4/3)


and it does not contain any points of S

Other than 1

⇒ 1 is not limit point of S

 Notes prepared by sanjeev kumar shukla


 9971245238 or 8750558444
z

Case 4 0<x<1 : x ∈ S

⇒ x = 1/n , n ∈ ℕ : n ≥ 2

Now n-1 < n < n+1 ⇒ > >

Then ( ) is a nbhd of x = 1/n and it

contains no point of S other than x = 1/n

⇒ x is not limit point of S.

Case 5 Let 0<x<1, x ∉ S

Let m and m+1 be consecutive naturals such


that m < < m+1 ⇒ ) is a nbhd of x,

And contains no point of S.

⇒ x is not limit point of S

⇒ Sl = {0}

Interior point - A point x ∈ ℝ is said to be an interior point


of a set s ⊆ℝ if ∃a nbhd V of x such that v ⊆ S

Isolated point – A point x ∈ S is said to be an isolat ed point of


S if x is not an interior point of S.

Boundary point – A real number x is said to be boundary point


of A ⊆ ℝ if each nbhd of x contains points of A
as well as points of A c

Closure of a set – Let A ⊆ ℝ, then intersection of all closed


sets containing A is called closure of A,
and denoted as A

 Notes prepared by sanjeev kumar shukla


 9971245238 or 8750558444
z

Chapter – 11.1 (Guidelines Questions)

(Q1) If x ∈ (0, 1) and Ԑx = min {x, 1-x} if |u-x| < Ԑx , show


that u ∈ (0, 1)

Solution |u-x| < Ԑx ⇒ x- Ԑx < u < x + Ԑx ….. (1)

Now Ԑx = min {x, 1-x } ⇒ Ԑx ≤ x and x + Ԑx ≤ 1 …. (2)

From (1) & (2) 0≤x- Ԑx<u< x + Ԑx ≤ 1

⇒ 0<u<1 ⇒ u ∈ (0, 1)

(Q2) show that the intervals (a, ∞) and ( -∞, a) are open sets,
and that [b,∞) and (-∞b] are closed sets

Solution:

1. Let x ∈ (a, ∞)

Let Ԑx < x – a then a < x- Ԑx < x < x + Ԑx< ∞

⇒ (x- Ԑx , x + Ԑx) ⊆ (a, ∞)

⇒ (a, ∞) is open set

2. Let x ∈ (-∞, a)

⇒ x < a, Let Ԑx < a- x

⇒ x + Ԑx < a

⇒ x- Ԑx < x < x + Ԑx< a

⇒ - ∞ < x - Ԑx < x < x + Ԑx < a

⇒ (x - Ԑx , x + Ԑx) ⊆ (-∞, a)

⇒ (-∞, a ) is open

 Notes prepared by sanjeev kumar shukla


 9971245238 or 8750558444
z

3. [b,∞)c = (-∞,b) which is open set ⇒ [b, ∞) is closed

(-∞, b ]c = (b, ∞) which is open set

⇒ (-∞, b] is closed set .

(Q3) See the Proof of open set propert part (b) on


page no.(5)&(6)

(Q4) Show that (0, 1] = then 0 < x ≤ 1 < 1 + 1/n ∀ n ∈ ℕ

⇒ x ∈ (0, 1 + 1/n) ∀ n ∈ ℕ

⇒ x ∈ ⋂ ( )

⇒ (0, 1] ⊆ ⋂ ( )

Now we’ll show that no other reals belong to


⋂ (0,1+1/n)

i.e if x < 0 then x ∉ ⋂ (0,1+1/n)

and if x > 1 then x ∉ ⋂ (0,1+1/n)

Case -1 if x < 0, then x ∉ ⋂ (0,1+1/n) ∀ n ∈ ℕ

⇒ x ∉ ⋂ (0,1+1/n)

Case – 2 if x > 1, then x-1 > 0 ⇒ > 0

⇒ ∃ n ∈ ℕ : n >

⇒ < x-1 ⇒ 1+ < x

⇒ x ∉ (0,1+1/n) for some n ∈ ℕ

 Notes prepared by sanjeev kumar shukla


 9971245238 or 8750558444
z

⇒ x ∉ ⋂ (0,1+1/n)

⇒ ⋂ (0,1+1/n) = (0, 1]

(Q5) See on page (22) Example

(Q6) See page (22) Example , S= { ℕ } then Sl = {0}

Here A = S U {0} ⇒ Al = {0} and {0} ⊆ A ⇒ Al ⊆ A

⇒ A is closed…

Chapter - 2.2 (R.G Bartle) Exercises continued from page


14.

(Q2) if a, b ∈ ℝ , then |a+b| = |a| + |b| ⟺ ab ≥ 0

Solution: |a+b| = |a| + |b|

⟺ |a+b|2 = (|a| + |b|)2

⟺ (a + b)2 = |a|2 + |b|2 + 2|a||b| (∵ |x|2 = x2)

⟺ a2 + b2 +2ab = a2 + b2 + 2|ab| (∵|x|2 = x2 )

⟺ 2ab = 2|ab| (left cancellation law of


addition)

⟺ ab = |ab| (left cancellation law of


addition)

⟺ ab ≥ 0

(Q4) |x-a| < Ԑ ⟺ -Ԑ < x-a < Ԑ ⟺ a-Ԑ < x < a+Ԑ

 Notes prepared by sanjeev kumar shukla


 9971245238 or 8750558444
z

(Q3) if x, y, z ∈ ℝ x ≤ z , show that x ≤ y ≤ z ⟺ |x-y| + |y-z| =


|x-z|

Solution: first let x ≤ y ≤ z (we’ll show |x -y| + |y-z| = |x-z|)

As x ≤ y ⇒ x-y ≤ 0 ⇒ |x-y| = y-x ….. (1)

y ≤ z ⇒ y-z ≤ 0 ⇒ |y-z| = z–y …. (2)

x ≤ z ⇒ x-z ≤ 0 ⇒ |x-z| = z-x …. (3)

from (1) and (2)

|x-y| + |y-z| = z - x = |x-z| … (by 3)

Conversely assume

|x-y| + |y-z| = z - x = |x-z| …. (*)

( We’ll show x ≤ y ≤ z )

As x ≤ z ∴ it is enough to show y ≮ x and y ≯ z

Let if possible y < x ⇒ x – y > 0 ⇒ |x-y| = x- y

Also y < x ≤ z ⇒ y < z ⇒ |y -z| = z – y

∴ |x - y| + |y - z| ⇒ x + z – 2y ≠ |x - z|

∴ we have a contradiction to (*)

⇒ y ≮ x ⇒ x ≤ y …. (a)

Similarly we can show that z ≮ y

∴ y ≤ z …. (b)

From (a) & (b) x ≤ y ≤ z

 Notes prepared by sanjeev kumar shukla


 9971245238 or 8750558444
z

(Q5) if a < x < b and a < y < b then then |x -y| < |b-a|

Solution a < x < b ⇒ a <x , y < b

a < y < b ⇒ a <x , -b < -y

⇒ a – b < x- y ….. (1)

Also a < y , x < b

⇒ -a > -y x < b ⇒ -a > -y & b > x ⇒ b – a > x – y


…. (2)

From (1) & (2) -(b - a) < x – y < b – a

⇒ |x - y| < b – a

(Q6) (a) |4x - 5| ≤ 13

⇒ - 13 ≤ 4x – 5 13

⇒ -13 + 5 ≤ 4x ≤ 13 +5

⇒ -8 ≤ 4x ≤ 18 ⇒ (-2 ≤ x ≤ 9/2)

(b) |x2 - 1| ≤ 3

⇒ |x2 - 1|2 ≤ 32

⇒ (x2 - 1)2 - 32 ≤ 0

⇒ (x2 – 1+3 ) (x2 – 1 - 3) ≤ 0

⇒ (x2 + 2 ) (x2 - 4) ≤ 0

⇒ as x2 + 2 > 0 ⇒ x2 - 4 ≤ 0 ⇒ |x|2 ≤ 4 ⇒ |x| ≤ 2

⇒ - 2 ≤ x ≤ 2
 Notes prepared by sanjeev kumar shukla
 9971245238 or 8750558444
z

(Q7) |x + 1| + |x - 2| = 7

We have following cases (1) x < -1, (2) -1 < x <2, (3) x > 2

(1) x < -1 ⇒ x < 2

⇒ x + 1 < 0 and x -2 < 0

⇒ |x +1| = -(x + 1) and |x -2| = 2 – x

⇒ |x +1| + |x - 2| = - x – 1 + 2 – x = -2x +1

∴ - 2x + 1 = 7 ⇒ x = -3

(2) -1 < x <2

⇒ |x + 1| + x + 1, |x - 2| = 2 – x

⇒ x + 1 + 2- x = 7 ⇒ 3 = 7 (an absurd)

(3) x > 2 ⇒ x > -1

∴ x + 1 + x -2 = 7 ⇒ 2x = 8 ⇒ x = 4

(Q8) (a) |x - 1| > |x + 1| ⇒ (x-1)2> (x+1)2

⇒ (x-1)2 - (x+1)2> 0

⇒ ((x - 1)+(x + 1))(x – 1- x -1) > 0

⇒ 2x (-2) > 0 ⇒ -4x > 0 ⇒ x < 0

∴ x ∈ (-∞, 0)

 Notes prepared by sanjeev kumar shukla


 9971245238 or 8750558444
z

(b) |x| + |x +1| < 2

Case – 1 (x) < -1 ⇒ x < 0 ⇒ |x| = -x

And |x + 1| = -x – 1

∴ -x – x – 1 < 2 ⇒ x > -3/2

Case -2 0 < x

⇒ |x| = x, |x +1| = x + 1

⇒ 2x + 1 < 2 ⇒ x < ½

Case -3 -1 < x <0

⇒ -x + x + 1 < 2 ⇒ (1 < 2)

∴ -3/2 < x < ½

(Q9) y = |x| - |x - 1|

x < 0

⇒ y = -x +x – 1 ⇒ y = -1

0 < x < 1

⇒ y = x + x -1 ⇒ y = 2x -1

x > 1 ⇒ y = x – x +1 ⇒ y = 1

also y(0) = -1 y(1) = 1

(Q10) 4 < |x + 2| + |x - 1| < 5

Case – 1 x < -2 ⇒ 4 < -x -2 – x +1 < 5

⇒ 4 < -2x – 1 < 5 ⇒ 5 < -2x < 6 ⇒ -5/2 > x > -3

 Notes prepared by sanjeev kumar shukla


 9971245238 or 8750558444
z

Case - 2 -2 < x < 1

⇒ 4 < x + 2 + x -1 < 5 ⇒ 4 < 2x +1 < 5

⇒ 3/2 < x < 2

∴ x ∈ (3/2, 2) U (-3, -5/2)

(Q12) (a) |x| = |y| ⇒ x2 = y2 ⇒ y = ± x

(b) |x| + |y| = 1

x ≥ 0, y ≥ 0 ⇒ x + y =1

x < 0, y < 0 ⇒ -x – y = 1

x < 0, y > 0 ⇒ -x + y = 1

x > 0, y < 0 ⇒ x – y = 1

(c) |xy| = 2 ⇒ xy = ± 2

(d) Do yourself

(Q13) Do yourself

(Q15) as a ≠ b ⇒ |a - b| > 0, Let Ԑ =

Then U = VԐ(a) is Ԑ- nbhd of a

V = VԐ(b) is Ԑ - nbhd of b

(we’ll show U ⋂ V = ∅ )

Let if possible U ⋂ V ≠ ∅ ⇒ ∃ x ∈ U ⋂ V

⇒ x ∈ U and x ∈ V

⇒ x ∈ VԐ(a) , x ∈ VԐ(b)

 Notes prepared by sanjeev kumar shukla


 9971245238 or 8750558444
z

⇒ |x - a| < Ԑ, |x - b| < Ԑ …. (1)

Now |a- b| = |a – x + x - b|

⇒ |a- b| ≤ |x-a| + |x-b|

⇒ |a-b| < Ԑ + Ԑ ⇒ |a-b| < 2Ԑ ⇒ |a-b| < 2

⇒ (3 < 2) which is a contradict ion

∴ U ⋂ V ≠ ∅ is not possible ⇒ U ⋂ V = ∅

(Q16) (a)

Case – 1 a > b ⇒ max {a, b } = a …. (1)

Also a > b ⇒ |a-b| = a-b

⇒ ½{a +b + |a-b|} = 1/2 {a+b+a-b} = a …… (2)

From (1) & (2) max {a, b} = 1/2{a +b +|a -b|}

Case - 2 a < b ⇒ max {a, b} = b … (3) and |a -b| = b –a

Also ½{a+b+|a-b|} = ½ {a+b+b-a} = b … (4)

From (3) & (4) max {a, b} = ½{a+b+|a -b|},


similarly when a = b

∴ max {a, b } = ½{a+b+|a -b|} ∀ a, b ∈ ℝ

And for min {a, b} = ½{a + b - |a-b|}

Consider case - 1 a > b ⇒ min {a, b} = b … (1)

Also a > b ⇒ |a-b| = a- b

⇒ ½{a +b - |a-b|} = ½{a+b +b-a} = b … (2)

 Notes prepared by sanjeev kumar shukla


 9971245238 or 8750558444
z

From (1) & (2) min {a, b} = ½{a + b - |a-b|}

Case – 2 a < b ⇒ min {a, b} = a … (3)

Also a < b ⇒ |a - b| = b – a

⇒ ½{a+b-|a-b|} = ½{a+b+a – b} = a … (4)

From (3) and (4) min {a, b} = 1/2 {a+b - |a-b|}

Case – 3 a = b ⇒ min {a, b} = a … (5) (∵ a = b)

½{a+b-|a-b|} = ½{a+a- |a-a|} (∵ a = b)

= ½{2a} = a … (6)

From (5) & (6) min {a, b} = ½{a+b -|a-b|}

(Q14) for VԐ(a) ⋂ Vδ(a) Let r = min {Ԑ, δ} (then we’ll show
VԐ(a) ⋂ Vδ(a) = Vr(a) )

Let x ∈ VԐ(a) ⋂ Vδ(a)

⇒ x ∈ VԐ(a) & x ∈ Vδ(a)

⇒ |x – a| <Ԑ And |x - a| < δ ⇒ |x-a| < min {Ԑ, δ} = r

⇒ |x-a| < r ⇒ x ∈ Vr(a)

⇒ VԐ(a) ⋂ Vδ(a) ⊆ Vr(a) … (1)

Let x ∈Vr(a) ⇒ |x-a| < r = min {Ԑ, δ}

⇒ |x – a| <Ԑ And |x - a| < δ

⇒ x ∈ VԐ(a), x ∈ Vδ(a) ⇒ x ∈ VԐ(a) ⋂ Vδ(a)

⇒ Vr(a) ⊆ VԐ(a) ⋂ Vδ(a) …. (2)

From (1) & (2) VԐ(a) ⋂ Vδ(a) = Vr(a)


 Notes prepared by sanjeev kumar shukla
 9971245238 or 8750558444
z

For VԐ(a) U Vδ(a) Let r = max{Ԑ, δ} (then we’ll show


VԐ(a) U Vδ(a) = Vr(a) )

Let x ∈ VԐ(a) U Vδ(a) ⇒ x ∈ VԐ(a) or x ∈ Vδ(a)

⇒ |x – a| <Ԑ And |x - a| < δ ⇒ |x-a| < max {Ԑ, δ} = r

⇒ x ∈ Vr(a) ⇒ VԐ(a) ⋂ Vδ(a) ⊆ Vr(a) …. (3)

Let x ∈ Vr(a) ⇒ |x-a| < r = max {Ԑ, δ} ⇒ |x – a|


<Ԑ or |x - a| < δ

⇒ x ∈ VԐ(a) or x ∈ Vδ(a)

⇒ x ∈ VԐ(a) U Vδ(a) ∴ Vr(a) ⊆ VԐ(a) ⋂ Vδ(a) …. (4)

From (3) & (4) VԐ(a) U Vδ(a) = Vr(a).

Section 2.3 –

Definition - Let S be a non-empty subset of ℝ

(a) The set S is said to be bounded above if there exists a


number u ∈ ℝ such that s ≤ u ∀ s ∈ S.

Each such number u is called an upper bound of S.

(b) The set S is said to be bounded below if there exists a


number w ∈ ℝ such that w ≤ s ∀ s ∈ S.

Each such number w is called a lower bound of S.

(c) A set is bounded if it is both bounded above and bounded


below.

 Notes prepared by sanjeev kumar shukla


 9971245238 or 8750558444
z

Definition Let S be a non - empty subset of ℝ

(a) If S is bounded above then a number u is said to be a


supremum (or a least upper bound)

Of S if it satisfies the conditions.

1. U is an upper bound of S, and


2. If V is any upper bound of S, then u ≤ v.

(b) If S is bounded below, then a number w is said to be


infimum of S (or greatest lower bound of S)

If is satisfies the conditions.

1. W is lower bound of S, and


2. If t is any lower bound of S, then t ≤ w.

Order Completeness property – (Section 2.4)

Every non- empty bounded above set has a supremum .

Archimedian property –

Let x ∈ ℝ , then ∃ nx ∈ ℕ : nx> x

Proof – Let if possible n ≤ x ∀ n ∈ ℕ, for some x ∈ ℝ,

⇒ ℕ is bounded above

⇒ By O.C.P ℕ has supremum say M

Then n ≤ m ∀ n ∈ ℕ ⇒ n +1 ≤ m ∀ n ∈ ℕ

 Notes prepared by sanjeev kumar shukla


 9971245238 or 8750558444
z

⇒ n ≤ m-1 ∀ n ∈ ℕ

⇒ m-1 is an upper bound of ℕ

⇒ m ≤ m-1 ⇒ 0 ≤ -1 which is not possible

⇒ ∃ nx ∈ ℕ : nx> x.

Chapter (2.3) & (2.4) (R.G Bartle) (Guideline’s Questions)

(Q1) S1 = {x ∈ ℝ : x ≥ 0}

1. Clearly 0 ≤ x ∀ x ∈S1 ⇒ 0 is a lower bound of S 1


2. We’ll show that 0 is inf S 1, for that we only need to
show that 0 is greatest lower bound of S 1

Let u be any lower bound of S 1

To show that - u ≤ 0 , let if possible , u > 0 then u ∈S 1


also 0 < u ⇒ ∃ x ∈ ℝ : 0 < x < u

⇒ x ∈S1 and x < u , which is a contradiction as u is a


lower bound of S 1

∴ u > 0 is not possible ⇒ u ≤ 0 ⇒ Inf S 1 = 0

3. We’ll now show that S1 is not bounded above.

Let if possible S 1 is bounded above then by order


completeness property S1 has supremum. Let

Sup S1 = m , Now clearly m > 0

⇒ m +1 > 0 ⇒ m+1 ∈S1

As Sup S1 = m ⇒ m +1 < m

 Notes prepared by sanjeev kumar shukla


 9971245238 or 8750558444
z

⇒ (1< 0) (which is a contradiction)

∴ S1 is not bounded above.

(Q2) Do yourself.

(Q3) S3 = {1/n : n ∈ ℕ}

Sup S3 = 1 ; clearly x ≤ 1 ∀ x ∈ S3

⇒ 1 is an upper bound of S3

Now let u be any upper bound of S 3

We’ll show 1 ≤ u ; let if possible 1 > u ;

Now 1 ∈S3 and 1 > u is a contradiction As u is upper bound of


S3

⇒ 1 > u is not possible ⇒ 1 ≤ u

⇒ Sup S3 = 1

(Q4) S4 = {1-(-1)n/n | n ∈ ℕ} find sup S4 and Inf S4

Solution S4 = {1+1, 1 – 1/2, 1+1/3, 1-1/4…….}

= {2, 0.5, 1.333, 0.75, 1.2, …..}

Sup S4 = 2 and Inf S4 = 0.5

 Notes prepared by sanjeev kumar shukla


 9971245238 or 8750558444
z

(Q5) Let S be a non-empty subset of ℝ that is bounded below,


show that

InfS = - Sup{-s : s ∈ S}

Proof – Let T = {-s : s ∈ S } and SupT = m

To show that - Inf S = -m

Claim – 1 –m is a lower bound of s

Let x ∈ S be any element then –x ∈ T ⇒ -x ≤ m (∵ m =


SupT)

⇒ x ≥ -m ∀ x ∈ S

⇒ -m is a lower bound of S

Claim – 2 –m is greatest lower bound of S

Let u be any lower bound of S

⇒ u ≤ x ∀ x ∈ S

⇒ -u ≥ -x ∀ -x ∈ T

⇒ -u is an upper bound of T, As m = supT

⇒ m ≤ -u -m ≥ u

⇒ -m is greatest lower bound of S

⇒ inS = -m = -SupT = - sup{-s : s ∈ S}

Note – Since S is bounded below ⇒ ∃ k ∈ ℝ : k ≤ x ∀


x ∈ S

 Notes prepared by sanjeev kumar shukla


 9971245238 or 8750558444
z

Now ∀ y ∈ T , -y ∈ S

⇒ -y ≥ k ⇒ y ≤ -k ∀ y ∈ T

⇒ T is bounded above and hence by order


completeness property T has a supremum, which we
called

As m, in above Proof.

(Q6) If a set S ⊆ ℝ contains one of its upper bounds, show


that this upper bound is the supremum of S.

Solution: Let u be upbd of S : u ∈ S (T.S.T Sup(S)= u)

As u is already an upper bound

∴ we only need to show u is least upbd

Let v be any up bd of S

As V is upbd of S (T.S.T u ≤ v)

And u ∈ S ∴ u ≤ v (By definition of upbd)

∴ Sup(S) = u

(Q7) (⟶) Let u = upbd of S , Let t ∈ ℝ and t > u (T.S.T t ∉ s)

Let if possible t ∈ s as u = upbd of S ⇒ t ≤ u which is


a contradiction as t ≰ u (∵ t > u)

∴ t ∉ s

(⟵) Let u ∈ ℝ be :

If t ∈ ℝ , t > u than t ∉ s (T.S.T u = upbd of S)

 Notes prepared by sanjeev kumar shukla


 9971245238 or 8750558444
z

Let s ∈ S (T.S.T s ≤ u)

Let if possible s ≰ u ⇒ s > u now S ⊆ ℝ ⇒ s ∈ S ⊆ ℝ

⇒ s > u and s ∈ ℝ by our hypothesis s ∉ S, which is


not possible as s ∈ S

⇒ s ≰ u is not possible

⇒ s ≤ u ∀ s ∈ S ⇒ u = upbd of S.

(Q9) A, B are bounded Subsets of ℝ, Show that A U B is


bounded and

Sup(A U B) = Sup {SupA, SupB}

Solution A, B are bounded ⇒ ∃ k, K ∈ ℝ and kl , Kl ∈ ℝ :

K ≤ a ≤ K , kl ≤ b ≤ Kl ∀ a ∈ A, b ∈ B

Now x ∈ A U B ⇒ x ∈ A or ∈ B

⇒ k ≤ x ≤ K or kl ≤ x ≤ Kl

⇒ min {k, kl } ≤ x ≤ max {K, Kl } ∀ x ∈ A U B

⇒ A U B is bounded

Now x ∈ A U B ⇒ x ∈ A or x ∈ B ⇒ x ≤ sup(A) or x ≤
Sup(B)

⇒ x ≤ Sup {Sup(A), Sup(B)} ….. (1)

Now Let u be any upper bound of A U B(We’ll


show Sup{Sup(A), Sup(B)} ≤ u )

∀ a ∈ A as A ⊆ A U B ⇒ a ∈ A U B ⇒ a ≤ u ∀ a ∈ A
….(2)

 Notes prepared by sanjeev kumar shukla


 9971245238 or 8750558444
z

∀ b ∈ B as B ⊆ A U B ⇒ b ∈ A U B ⇒ b ≤ u ∀ b ∈ B
….(3)

From (2) & (3) ⇒ u is and upbd of A and u is an


upbd of B

⇒ Sup(A) ≤ u and Sup(B) ≤ u

⇒ Sup {Sup(A), Sup(B)} ≤ u … (4)

From (1) & (4)

⇒ Sup(A U B) = Sup{Sup(A), Sup(B)}

(Q10) Let S be bounded set in ℝ and S ≠ ∅

Let S0 ⊆ S and S0 ≠ ∅ then

Inf (S) ≤ Inf(S0) ≤ Sup(S0) ≤ Sup(S)

Solution - ∀ x ∈S0 ⊆ S

⇒ x ∈ S ⇒ Inf(S) ≤ x ∀ x ∈S 0

∈ Inf (S) is a lower bound of S 0

⇒ Inf(S) ≤ Inf(S0)….. (1)

Again, ∀ x ∈S0 , x ∈ S

⇒ x ≤ Sup(S) ∀ x ∈S0

⇒ Sup(S) is an upbd of S 0

⇒ Sup(S0) ≤ Sup(S) …. (2)

Now let s0 ∈ S0 (be fixed then)

Inf(S0) ≤ s0 Sup(S0) ⇒Inf(S0) ≤ Sup(S0) …. (3)

 Notes prepared by sanjeev kumar shukla


 9971245238 or 8750558444
z

From (1), (2) and (3) ⇒ Inf (S) ≤ Inf(S 0) ≤ Sup(S0) ≤


Sup(S)

(Q11) Let S ⊆ ℝ and suppose s* = Sup(S), s* ∈ S, If u ∉ S ,


show that

Sup{S U {u}} = Sup{ s* , u}

Solution

Case – 1 s* < u , ⇒ Sup{ s* , u} ≤ u

Claim – Sup{S U {u}} = u

1. Let x ∈ S U {u} ⇒ x ∈ S or x = u ⇒ x ≤ s * or x =
u ⇒ x < u or x = u ⇒ x ≤ u
2. Let K be any upper bound of S U {u} (T.S.T u ≤ K)

As K is upbd of S U {u} ⇒ x ≤ K ∀ x ∈S U {u} as u


∈S U {u} ⇒ u ≤ K

⇒ Sup {S U {u} } = u = Sup{ s* , u}

Case – 2 If s* > u then Sup{ s*, u } = s*

We’ll claim – Sup{ S U {u}} = s* = Sup{ s*, u}

Claim – 1 s* is an upper bound of S U {u}

As s* = SupS and s* > u

⇒ x ≤ s* ∀ x ∈ S and u < s*

⇒ x ≤ s* ∀ x ∈ S U {u}

⇒s* is an upper bound of S U {u}

Claim – 2 s* is least upper bound of S U {u}

 Notes prepared by sanjeev kumar shukla


 9971245238 or 8750558444
z

Let k be any upper bound of S U {u} , then x ≤ k ∀ x


∈S U {u} …. (1)

Note - we are given that s* = SupS ∈ S

⇒ s* ∈ S ⊆S U {u}

Use this fact in (1) we get s* ≤ K ⇒ s* is least


upper bound of S U {u}

⇒ Sup{ S U {u}} = s* ⇒ Sup{ S U {u}} = Sup{ s*, u}.

(Q12) Show that a non - empty subset which is finite, in ℝ


contains it’s supremum.

Proof - Let S be any finite set containing n elements we’ll


prove the resukt by applying induction on

elements in S.

when s contains a single element i.e n = 1

Let S = {a1} then clearly Sup{s} = a1 ∈ S

Now when S contains two elements say a 1 & a2

(W.L.O.G) Assume , a1 > a2 then Sup{ a1 , a2 } = a1 ∈


S;

Now let us assume that result is true for all sets


containing K elements; we’ll show that result is also
true for n = K + 1 i.e for a set containing (K + 1)
elements
Let S = { a1 , a2 , ….. ak , ak+1 }

 Notes prepared by sanjeev kumar shukla


 9971245238 or 8750558444
z

Then S = Sl U { ak+1 } , where Sl = { a1 , a2 , ….. ak ,


ak+1 }

Note – since O(S) = k + 1 ⇒ ai ≠ aj ∀ i ≠ j

Now as Sl is a set containing k elements ∴ by our


assumption Sup Sl ∈ Sl , let Sup Sl = s*

Now by the Proof of above question (2)

We see SupS = Sup{ s* , ak+1}

⇒ SupS = s* or SupS = ak+1

Case -1 if SupS = s* ∈Sl ⊆ S

⇒ SupS ∈ S,

Case – 2 SupS = ak+1 ∈S (∵ S = Sl U { ak+1 } )

⇒ SupS ∈ S, ∴ in both of the cases .

⇒ SupS ∈ S

(Q15)Write down details in Proof of the following


theorem.

“A number u is the supremum of a non - empty subset S of


ℝ iff u satisfies conditions

1. S ≤ u ∀ s ∈ S, 2. If v < u , then there exists s * ∈ S


such that v < s* ”.

 Notes prepared by sanjeev kumar shukla


 9971245238 or 8750558444
z

Proof – Forward part – Assume u = SupS

⇒ S ≤ u ∀ s ∈ S, if v < u, as u is SupS ⇒ u is l.u.b and


hence v is not an upper bound

⇒ ∃ s* ∈ S : v < s*

Conversely assume that – 1 & 2 hold

Then 1 ⇒ u is an upper bound of S also if v is any upper


bound of S

T.S.T u ≤ v , let if possible v < u, then by condition 2 ∃ s *


∈ S : s* > v , which is a contradiction as v is upper bound
of S.

⇒ v < u is not possible ⇒ u ≤ v

⇒ SupS = u

Supremum and Infimum of real valued functions defined


on any region.

Let D be any region in space (Note – Space stands for ℝ,


ℝ2 , ℝ3,…..ℝn)

Let f : D ⟶ ℝ, be any function then Supf, the supremum of


the set f(D)

Defined as f(D) = {f(x)|x∈D}

i.e Supf = Supf(D)

Completeness property of ℝ -

Every non-empty bounded above subset of ℝ has a


supremum.

 Notes prepared by sanjeev kumar shukla


 9971245238 or 8750558444
z

Solved Questions Exercise 2.4

(Q1) Show that Sup {1- 1/n : n ∈ ℕ} = 1

Solution – Let x ∈ {1-1/n | n ∈ ℕ} be any element


then clearly x = 1-1/n < 1 , n ∈ ℕ

⇒ x < 1 ∀ x ∈ {1-1/n | n ∈ ℕ }

⇒ 1 is an upper bound of {1 -1/n | n ∈ ℕ }

Now let u be any upper bound of {1 -1/n | n ∈ ℕ }

To show that - 1 ≤ u , Let if possible 1 > u ⇒ 1 -u > 0 ⇒


> 0

By archimedian property there exists n ∈ ℕ : n > ⇒ 1-u

> 1/n

⇒ 1-1/n > u

Let x = 1-1/n; then x ∈ {1-1/n | n ∈ ℕ } and x > u ; which is a


contradiction as u is an

Upper bound of {1-1/n | n ∈ ℕ } ∴ our assumption that 1 > u is


wrong ⇒ 1 ≤ u

⇒ 1 = Sup{1-1/n | n ∈ ℕ }

(Q2) If S= { | m, n ∈ ℕ} , find InfS and SupS.

Solution – Let S1 = { | n ∈ ℕ } & S2 = { | m ∈ ℕ }

Then S = S1 + S2 and ⇒ Sup(S) = Sup(S1 ) + Sup(S2 )….. (A)

 Notes prepared by sanjeev kumar shukla


 9971245238 or 8750558444
z

Inf(S) = Inf(S1) + Inf(S2) …. (B)

Now Sup(S1) = 1 , Inf(S1) = 0

Sup(S2 ) = 0, Inf(S2) = -1

⇒ Sup(S) = 1, Inf(S) = -1

Note We’ll prove (A) & (B) later .

(Q3) Let S ⊆ ℝ ne non- empty. Prove that if a number


u in ℝ has the properties (i) for every n ∈
ℕthenumber u-1/n is not an upper bound of S, and
(ii) for every number n ∈ ℕ the number u+1/n us an
upper bound of S, then u = Sup(S), also conversely.

Solution – Claim – 1 u is an upper bound.

Let s ∈ S be any element , To show that – s ≤


u, Let if possible s > u

⇒ s – u > 0 ⇒ > 0 ∃ n ∈ ℕ : n >

⇒ 1/n < s – u ⇒ u + 1/n < s, s ∈ S

Which is a contradiction as u +1/n is an upper bound of S


and hence can’t be less than any element of S.

⇒ s ≤ u ⇒ u is upper bound of S.

Claim – 2 for any upper bound v of S. u ≤ v

Let if possible u > v ⇒ u – v > 0

⇒ > 0, ∃ n ∈ ℕ : n >

⇒ 1/n < u – v ⇒ v < u – 1/n

 Notes prepared by sanjeev kumar shukla


 9971245238 or 8750558444
z

Now ∀ x ∈ S , x ≤ v < u-1/n ⇒ x < u-1/n ∀ x ∈ S

⇒ u-1/n is an upper bound of S, which is a contradiction


as u-1/n can’t be an upper bound.

U > v is not possible ⇒ u ≤ v

⇒ Sup(S) = u

Conversely Let Sup(S) = u

⇒ x ≤ u ∀ x ∈ S ⇒ x ≤ u < u +1/n ∀ x ∈ s ∀ n ∈ ℕ

⇒ u +1/n is an upper bound of S ∀ n ∈ ℕ

Also ∀ n ∈ ℕ, u-1/n is not an upper bound (∵ u is


l.u.b and u-1/n < u ∀ n ∈ ℕ)

(Q4) Let S be any non – empty bounded set in ℝ;

(a) Let a > 0 and let aS = {as | s ∈ S}. Prove that inf(aS)
= a inf(S), Sup(aS) = a Sup(S).
(b) Let b < 0, and let bS = {bS |s ∈ S}. Prove that
inf(bS) = b Sup(S), Sup(bS) = b Inf(s).

Solution - (a) Claim – 1 a Inf(S) is a lower bound of ‘aS’

Let x ∈ aS be any e lement then x = as, s ∈ S.

Now Inf(S) ≤ s ∀ s ∈ S ⇒ aInf(S) ≤ as ∀ s ∈ S

⇒ aInf(S) ≤ x ∀ x ∈ aS

⇒ aInf(S) is a lower bound of aS.

Claim – 2 if V is any lower bound of aS then v ≤ aInf(S).

As v is a lower bound of aS, therefore v ≤ a s,


∀ s ∈ S

 Notes prepared by sanjeev kumar shukla


 9971245238 or 8750558444
z

⇒ v/a ≤ s ∀ s ∈ S ⇒ v/a is a lower bound of S

⇒ v/a ≤ Inf(S) ⇒ v ≤ a Inf(S)

⇒ from claim – 1 and claim – 2, we have

Inf(aS) = a Inf(S)

Now we’ll prove that Sup(aS) = aSup(S)

Claim – 1 a Sup(S) is an upper bound of aS.

Let x ∈ aS (be any) then x = as, s ∈ S …(1)

Also s ≤ Sup(S) ∀ s ∈ S ⇒ as ≤ aSup(S), ∀ s ∈ S … (2)

From (1) & (2) we have x ≤ aSup(S).

⇒ aSup(S) is an upper bound of aS.

Claim – 2 if V is any upper bound of aS, then aSup(S) ≤ v

As v is an upper bound of aS

⇒ v ≥ as ∀ s ∈ S

⇒ v/a ≥ s ∀ s ∈ S

⇒ v/a is an upper bound of S

And hence Sup(S) ≤ v/a

⇒aSup(S) ≤ v

And hence Sup(aS) = aSup(S)

(b) we’ll first prove Inf(bS) = bSup(S)

Claim – 1 bSup(S) is a lower bound of bS.

Let x ∈ bS ⇒ x = bs, s ∈ S …. (1)

 Notes prepared by sanjeev kumar shukla


 9971245238 or 8750558444
z

Now s ≤ Sup(S) ∀ s ∈ S ⇒ bs ≥ bSup(S) ∀ s ∈ S (∵ b < 0)


… (2)

From (1) & (2) ⇒ x ≥ bSup(S) ⇒ bSup(S) is lower bound


of bS.

Claim – 2 if v is any lower bound of bS, then bSup(S) ≥ v

As v is a lower bound of bS

⇒ v ≤ bs ∀ s ∈ S ⇒ v/b ≥ s ∀ s ∈ S

⇒ v/b is an upper bound of S

⇒ Sup(S) ≤ v/b ⇒ bSup(S) ≥ v

From claim – 1 & claim -2 , we have

Inf(bS) = bSup(S)

Now we’ll show that Sup(bS) = bInf(s).

Claim – 1 we shall show that –

bInf(S) is an upper bound of bS

Let x ∈ bS be any element then we have that , x= bs from


s ∈ S … (1)

Now as by definition of Inf(S), we have

Inf(S) ≤ s ∀ s ∈ S

⇒ bInf(S) ≥ bs ∀ s ∈ S … (2) (∵ b < 0)

From (1) & (2) we have

X ≤ bInf(S) ∀ x ∈ bS

⇒ x ≤ bInf(S) ∀ x ∈ bS

 Notes prepared by sanjeev kumar shukla


 9971245238 or 8750558444
z

Claim – 2 bInf(s) is least upper bound of bS.

Let v be any upper bound of bS

To show that – (bInf(S) ≤ v)

As v is upper bound of bS ⇒ v ≥ bs ∀ s ∈ S

⇒ v/b ≤ s ∀ s ∈ S ⇒ v/b is a lower bound of S

⇒ v/b ≤ Inf(s) ⇒ v ≥ bInf(S)

Hence proved.

Guideline Questions from Chapter 2.4 (Bartle)

(Q6) Let A and B be bounded non- empty sets in ℝ and

A + B = {a + b: a ∈ A, b ∈ B}.

Show that

(a) Sup(A + B) = sup(A) + Sup(B)

(b) inf(A + B) = Inf(A) + Inf(B)

Solution Let SupA = m1& SupB = m2 ….

(a) To show that – Sup(A+B) = m1 + m2

Claim – 1 m1 + m2 is an upper bound of A + B

Let x ∈ A + B (be any element)

⇒ x = a1 + b1 , a1 ∈ A, b1 ∈ B

Now a1 ≤ m1 , b1 ≤ m2

∴ x ≤ m1 + m2 ⇒m1 + m2 is an upbd of A + B

 Notes prepared by sanjeev kumar shukla


 9971245238 or 8750558444
z

Claim – 2 m1 + m2 is least upper bound

Let v be any upbd of A + B (T.S.T m 1 + m2 ≤ v )

⇒ a + b ≤ v ∀ a ∈ A, b ∈ B

⇒ b ≤ v – a ∀ b ∈ B, a ∈ A

⇒ v – a is na upbd of B ∀ a ∈ A

⇒ m2 ≤ v – a ∀ a ∈ A

⇒ a ≤ v - m2 ∀ a ∈ A ⇒ v - m2 is an upbd of A

⇒ m1 ≤ v - m2

⇒ m1 + m2 ≤ v

∴ m1 + m2 is l.u.b of A + B

⇒ Sup(A + B) = m1 + m2 = sup(A) + Sup(B)

(b) Let infA = m1 and infB = m2 (T.S.T inf(A+B) = m1 +


m2)

(1) then ∀ x ∈ A+B, x = a + b, a ∈ A, b ∈ B

⇒ x ≥ m1 + m2 (∵ a ≥ m1 , b ≥ m2 )

⇒ m1 + m2 is a lower bound of A + B

(2) Let v be any lower bound of A + B (m 1 + m2 ≥ v)

Then a + b ≥ v ∀ a ∈ A, b ∈ B ⇒ a ≥ v – b ∀ a ∈ A, ∀
b ∈ B

 Notes prepared by sanjeev kumar shukla


 9971245238 or 8750558444
z

⇒ m1 ≥ v – b ∀ b ∈ B ⇒ b ≥ v - m1 ∀ b ∈ B ⇒ v - m1 is
lower bound of B ⇒ m2 ≥ v - m1

∴ m2 + m1 ≥ v ⇒ inf(A+B) = m1 + m2

Q12 Given any x ∈ ℝ , show that there exists a unique


n ∈ ℤ such that

n – 1 ≤ x < n.

Q13 If y > 0, show that there exists n ∈ ℕ such that


< y

Q18 If u > 0, is any real number any x < y, show that


there exists a rational number r such that

x < ru < y.

Solution 12 x ∈ ℝ , Define E = {m ∈ ℕ: m > x} (then by A.P


E ≠∅)

Also E ⊆ ℕ, E ≠∅ ∴ By W.O.P , E has a least element


(unique)

Say n ∈ E ⇒ n > x … (I)

Also n - 1 ∉ E ( ∵ n is least element in E )

⇒ n – 1 ≤ x …. (II)

From (I) & (II) n – 1 ≤ x < n

Solution 13 As y > 0 ⇒ 1/y > 0 ∴ By A.P, ∃ n ∈ ℕ :


n > 1/y

∴ ∃ n ∈ ℕ : 1/n < y … (I)

 Notes prepared by sanjeev kumar shukla


 9971245238 or 8750558444
z

Now we know ∀ n ∈ ℕ, n < 2n ⇒ 1/2n < 1/n … (II)

From (I) & (II) 1/2n < 1/n < y ⇒ 1/2n < y , for
some n ∈ ℕ

Solution 18 x < y and u > 0 ⇒ 1/u > 0 ⇒ x/u < y/u

By density theorem ∃ r ∈ Q :

x/u < r < y/u ⇒ x < ru < y

Q14 Show that there exists a +ve real number y : y 2 = 3

Q15 If a > 0, then show that there exists a +ve real


number z : z2 = a

Q16 Show that there exists a +ve real number u such that
u3 = 2

Solution – 14 Do as question 15 be low (replace, z by y and a


by 3)

Solution 15. Define S = {S ∈ ℝ : s ≥ 0 and s2 < a}

Case – 1 0 < a < 1 Choose 0 < s < a , then s 2 < a2 < a


(∵ a < 1)

⇒ s2 < a also s > 0

⇒ s ∈ S ⇒ s ≠ ∅

Now we’ll show that S is bounded above Let s ∈ S (be


any) then s2 < a < 1 (∵ a < 1)

⇒ S is bounded above by 1

Case – 2 a > 1 , s ∈ S then s 2 < a and s > 0

 Notes prepared by sanjeev kumar shukla


 9971245238 or 8750558444
z

As a > 1 ⇒ 12 = 1 < a also 1 > 0 ⇒ 1 ∈ S ⇒ S ≠ ∅

And we’ll show S is bounded above by a.

Let if possible ∃ s ∈ S : s > a ⇒ s 2 > a2> a (∵ a > 1)

⇒ s2 > a …… (I)

But s ∈ S ⇒s2 < a …. (II)

(I) & (II) contradict each other ∴ S is bdd above by a

∴ in each case S is non empty bdd above set ∴ By


completeness property S has supremum

Let Sup(S) = Z, Claim - Z2 = a

Claim – 1 Z2 ≮ a , Let if possible Z 2 < a

(we’ll find m ∈ ℕ : z + 1/m ∈ S i.e (Z+1/m)2 < a, which


is certainly a contradiction as Sup(S) = Z )

Consider - (Z+1/m)2 = Z2 + 1/m2 + 2z/m

= Z2 + (2z + 1) (∵1/m2< 1/m)

Now (Z+1/m)2 < a if Z2 + (2z + 1) < a

⇒ if (2z + 1) < a - Z2

⇒ m > ( ) (∵ a - Z2 > 0 )

And such an m exists by A.P

∴ ∃ m ∈ ℕ : (Z+1/m)2 < a ⇒ z + 1/m ∈ S

As Sup(S) = Z ⇒ z + 1/m < Z

 Notes prepared by sanjeev kumar shukla


 9971245238 or 8750558444
z

⇒ 1/m ≤ 0

⇒ (1 ≤ 0) not possible ∴ Z2 ≮ a

Claim - 2 Z2 ≯ a , Let if possible Z2 > a

(we’ll find m ∈ ℕ : z - 1/m ∈ S i.e (Z - 1/m)2 < a )

Consider - (Z - 1/m)2 = Z2 + 1/m2 - 2z/m > Z2 -


2z/m

Now (Z - 1/m)2 > a if Z2 - 2z/m > a i.e if m > ( )

(∵ Z2 > a)

And such an m exists by A.P

∴ ∃ m ∈ ℕ : (Z - 1/m)2 > a Now Let y ∈ S (be any )

Then y2 < a < (Z - 1/m)2 ⇒ y < Z - 1/m ∀ y ∈ S


which is not possible as Sup(S) = Z is least up. Bd of S ∴
Z2 ≯ a and hence Z = a

 Notes prepared by sanjeev kumar shukla


 9971245238 or 8750558444
z

Sol. 16

S = { s ∈ ℝ : s > 0, s3 <2}

(1) Clearly S ≠ ∅
(∵ ∃ 1 >0 and 13=1<2⟹1∈ S)
(2) S is bounded above by 2
∵ If S is not bounded above by 2
⇒∃ s∈S:s>2 ⇒ s3>8 ............(i)
Now s∈S⇒s3<2 ............(ii)

(i)and (ii) contradict each other

∴ s is bounded above by 2.

∴ By completeness property S has a Supremum say u i.e


Sup(s)=u

Claim u3=2

Claim-1 u3<2

Let if possible u 3<2

(We’ll find m∈≤: ( ) < 2)

Consider ( ) = u3 + + + < 2

< u3 + + + (∵ u >1 ⇒u1>u)

‘ = u3+ )

Now ( ) < 2 If u3 + (1+6u2)<2

 Notes prepared by sanjeev kumar shukla


 9971245238 or 8750558444
z

⇒ m>( ) ( ∵ 2-u3> 0)

Such an m exists by A.P.

∴ ∃ m ∈ℕ : ( ) <2 ⇒ u + ∈ S

⇒ u+ ≰u (∵u=sup(s))

⇒ 1≤0 which is not possible ⇒ u3 ≮2

Claim-2 u3≯2

Let if possible u 3>2

(We’ll claim ∃ m∈ ℕ : ( ) > 2)

Consider ( ) = u3 – – +

>u3 – –

⇒ ( ) > u3 – –

Now m3>m ⇒ < ⇒ >( )

⇒ ( ) > u3 – –

⇒ ( ) > u3 – (1+3u2)

∴ ( ) > 2 If u3 - (1+3u2) >2

⇒ m >( ) ( ∵ u3-2≠0)

and such an m exists by A.P.

 Notes prepared by sanjeev kumar shukla


 9971245238 or 8750558444
z

∴ ∃ m ∈ℕ : ( ) > 2

Let s ∈ S ( be any )

then s3< 2 <( ) ⇒ s < u - ∀ s ∈ S

⇒ u - is an upper bound of S

As Sup(s) = u ⇒ u ≤ u-

⇒ 0 ≤ -1 which is not possible

∴ u3 ≯2

and hence by claim 1 and claim 2 u 3 = 2

Note : Q.E.D stand for “quod erat demonstration “ ,


which is Latin for “which was to be demonstrated”.

Section 2.4

Sol 6. If A and B are bounded subsets of ℝ

Then (1) Sup (A+B) : SupA + SupB

(2)Inf(A+B) : InfA + InfB

Where A+B : {a+b/a∈A,b∈B}

Proof. Let SupA = m1 and SupB=m2

To show that Sup(A+B) : m1+m2

Claim 1 : m1+m2 is an upper bound of A+B

 Notes prepared by sanjeev kumar shukla


 9971245238 or 8750558444
z

Let x∈A+B (be any element)

⇒x∈a1+b1;a1∈A , b1∈B ….(1)

Also, M1 = Sup(A) ⇒a≤m1 ∀a∈A

M2 = Sup(B) ⇒b≤m2 ∀b∈B

∴a1≤m1and b1 ≤ m2

⇒a1+b1≤M1+M2….(2)

From (1) and (2) :

⇒ a1+b1≤M1+M2 ∀x∈A+B

⇒ x≤M1+M2 ∀x∈A+B

M1+M2 are upperbound of A+B

Claim 2 : If v is any upper bound os A+B then M 1+M2 ≤ v

As v = upper bound of A+B

⇒ a_b≤v ∀ a∈A and ∀ b∈B

⇒ b≤v-a ∀ a∈A and ∀ b∈B

⇒v-a is an upperbound of B ∀a∈A

M2 ≤ v –a ∀a∈A(∵sup(B) <M2)

⇒a≤v-M2 ∀a∈A

⇒v-M2 is an upper bound of A

⇒M1≤ v-M2

⇒M1+M2≤v

 Notes prepared by sanjeev kumar shukla


 9971245238 or 8750558444
z

∴ Sup(A+B) : M1 +M2 = Sup(A) + Sup(B)

Sol 7. f,g :X→ℝ

To prove :
(i) Sup{f(x) +g(x) : x∈X} ≤ Sup{f(x):x∈X} +
Sup{g(x):x∈X}
(ii) Inf{f(x) +g(x) : x∈X} ≥ Inf{f(x):x∈X} + Inf{g(x):x∈X}

Sol (i) : Let Sup{f(x) : x∈X} = M 1 and Sup{g(x):n∈X}=M2

Claim : M1+M2 is an upper bound of {f(x)+g(x) : x∈X}

As : M1 = Sup{f(x):x∈X}

M2=Sup{g(x):x∈X} ⇒ g(x)≤M 2∀x∈X

⇒ f(x) +g(x) ≤ M1+M2 ∀x∈ℕ

M1+M2 is and upper bound of f(x)+g(x):x∈X}

⇒Sup{(f(x)+g(x)}:x∈X}

≤ M1+M2

Sup{f(x):x∈X} + Sup{g(x):x∈ℕ}

(ii): Let Inf{f(x) : x∈X} = M1 and Inf{g(x):n∈X}=M2

Claim : M1+M2 is an lower bound of {f(x)+g(x) : x∈X}

As : M1 = Inf{f(x):x∈X} ⇒f(x)≥M1∀x∈X

M2=Inf{g(x):x∈X} ⇒ g(x)≥M2∀x∈X

⇒ f(x) +g(x) ≥ M1+M2 ∀x∈ℕ

M1+M2 is and lower bound of f(x)+g(x):x∈X}

 Notes prepared by sanjeev kumar shukla


 9971245238 or 8750558444
z

⇒Inf{(f(x)+g(x)}:x∈X}

≥ M1+M2

Inf{f(x):x∈X} + Inf{g(x):x∈ℕ}

Sol 8 : X=(0,1)

Y=(0,1)

h:XXY → ℝ

h(x,y)=2x+y

a) ∀x∈X,find
f(x)=Sup{h(x,y):y∈Y}
then find Inf{f(x):x∈X}
b) ∀y∈Y,find
g(y)=Inf{h(x,y):x∈X}
then find Sup{g(y):y∈Y}

a) f(x) = Sup{h(x,y):y∈Y}=Sup{2x+y|y∈Y}
=2x+Sup{y:y∈Y}

=2x+Sup((0,1)) = 2x+1

⇒f(x)=2x+1 …….(1)

Inf{f(x):x∈X}=Inf{2x+1|x∈X}=Inf{2x+1:o<x<1}
=Inf{2x+1:1<2x<1<3}

=Inf{1,3********************

b) f(x) = Inf{h(x,y):x∈X}=Inf{2x+y:x∈(0,1)}
=y+Inf{2x:x∈(0,1)}

= y+Inf{0,2)=y+0=y}

 Notes prepared by sanjeev kumar shukla


 9971245238 or 8750558444
z

⇒g(y)=y

Now Sup{g(y):y∈Y}=Sup{y:y∈(0,1)}

=Sup(0,1)=1

Sol 9 : X=(0,1)=Y

h:XXY→ℝ

h(x,y)={

for any x∈X

a) f(x)=Sup{h(x,y):y∈Y}

=Sup({h(x,y):x<y}U{h(x,y):x≥y})

=Sup({0}U{1})=1

Inf(x):x∈X}=Inf{1:x∈X}=1

b) g(y)=Inf{h(x,y):x∈X}=Inf({h(x,y):x≥y}U{h(x,y)

=Inf({1}U{0})=Inf{1,0}=0

Sol 12 : x∈ℝ

prove that ∃ m∈ℤ (Unique)

n-1≤x≤n

Define E ={m∈ℕ:m>x}

(then by A.P. E≠∅)

Also, E⊆ℕ,E≠∅

By W.O.P , E has a least element of E

 Notes prepared by sanjeev kumar shukla


 9971245238 or 8750558444
z

⇒n>x……(1)

Now , n-1∉E (∵ n is least element of E)

⇒n-1≤x…..(2)

From (1) and (2)

⇒ n-1≤x<n

Note : Since least elements of a set is unique ,∴ n is unique

m-1 ≤x<n

Sol 13 : As y>0

⇒ >0

∴ By A.Ap ∃ n∈ℕ

n>

⇒ <y …………....(1)

Now we know ∀n∈ℕ ;

⇒n<2n

< ……….(2)

From (1) and (2)

< <y

⇒ < y

Sol 15 : < < y

 Notes prepared by sanjeev kumar shukla


 9971245238 or 8750558444
z

To show that ∃ z>0 :

z2 =a ,a>0

Define:

S={s∈ℝ:s≥0 and s2<a}

Case 1 : 0<a<1

Choose ; 0<s<a

Then , s2<a2<a (∵a<1)

⇒s∈S

⇒s≠∅

Now we’ll show that :

S is Bounded above

Let s∈S be any element

⇒s2<a<1

⇒s2<1 ⇒ s<1

S is bounded above

By completeness property S has a supremum

Let Sup(s)=z

Now, for any s∈S

s2<a<( ) (by (1))

 Notes prepared by sanjeev kumar shukla


 9971245238 or 8750558444
z

⇒s2<( ) ∀s∈S

⇒s <( ) ∀s∈S

⇒z- is and U.B. of S

⇒z≤z-= (∵ Sup(s) = z)

⇒0≤ ⇒ 0≤-1 ⇒ 0≥1

∴ which is an contradiction

∴z2>a is not possible

⇒z2≯a

By claim 1 and claim 2 we have

z≮a and z2≯a

⇒ z2=a

Case 2: S={S∈ℝ|S>0,S2≤a}

If a>1, then, 1>0

And 12 = 1 ≤a

⇒1∈S

⇒s≠∅

Now we’ll show S is bounded above by a

Let id possible ∃ spme s∈S:

s>a ⇒ s2>a2………….(1)

 Notes prepared by sanjeev kumar shukla


 9971245238 or 8750558444
z

Now,s∈S

⇒s2<a…………(2)

From (1) and (2) :

a2<s2<a

⇒a2<a which is not possible

(∵a>1 ⇒ a2>a)

∴∄s∈S : s>a

⇒ s≤a ∀s∈S

⇒ S is bounded above by a

∴ By completeness property, S has a supremum say


Sup(s)=z

Claim 1: z2≮a

Let if possible: z2<a

We’ll find m∈ℕ:

z+ ∈ S i.e. ( ) < a

CONS :

( ) = z2 + + < z2+ (2z+1) ( ∵ < )

Now, ( ) <a If z2 + (2z+1) <a

 Notes prepared by sanjeev kumar shukla


 9971245238 or 8750558444
z

i.e. If )< a - z 2

⇒ m> ((2z+1) / (a-z2)) (∵a-z2>0)

Such an m exists by A.P.

⇒∃m∈ℕ:

( z + )2 < a

⇒z+ ∈S

As, Sup(s)=z

⇒ z+ ≤ z

⇒ <0

⇒1≤0 which is a contradiction ⇒ z 2<a is not possible

⇒z2 ≮ a

Claim 2 : z2 ≯a

Let if possible : z2> a

We’ll find m∈ℕ

( z + )2 > a

CONS. :

( z + )2 = z 2 + 1 / m 2 – 2 z / m > z 2 -

N o w , ( z + )2 > a I f z 2 - >a

 Notes prepared by sanjeev kumar shukla


 9971245238 or 8750558444
z

i.e. If < z2-a ⇒m > (2z/z2-a) (∵z2-1)

and such an m exists by A.Pa

∴ m ∈ ℕ : ( z + )2 > a

Sol 16 : S={s∈ℝ:s>0;s3<2}

(1) Clearly, s≠∅


( ∵∃1>0 and 13=1<2
⇒1∈S)

Also, S is bounded above by 2

If S is not bounded above by 2

⇒∃s∈SLs>2 ⇒ s3>8…….…...(1)

Now, s∈S ⇒ s3<2………..(2)

Thus (1) and (2) are contrary to each other

∴∄ s∈S : s>2 ⇒ s≤2 ∀s∈S

∴ S≠∅ is bounded above

∴ By completeness property, S has a supremum

Let Sup(S) = u

Claim u3=2

Claim 1: U3≮2

Let if possible U3<2

We’ll claim ∃m∈ℕ, such that (u+ )3 <2

 Notes prepared by sanjeev kumar shukla


 9971245238 or 8750558444
z

Consider (u+ )3 = u3 + 1/m3 + 3u2/m+ 3u/m2<2

< u3 + 1/m + 3u2/m+ 3u2/m (∵u>1 ⇒ u2>u)

= u3 + 1/m ( 1+3u2+3u2)

= u3 + 1/m ( 1+6u2)

Now, (u+ )3<2 If u3+ (1+6u2) < 2

⇒ (1+6u2) < 2-u3

⇒m > (1+6u2) / (2-u3) (∵2-u3>0)

Such an m exists by A.P.

∴∃m∈ℕ:

(u+ )3< z

⇒(u+ )∈ S

u+ ≤ u

⇒u+ ≤u

⇒ < 0 ⇒ 1≤0 which is a contradiction

⇒u3<2 is not possible

Claim 2: u2≯2

Let if possible u 3>2

We’ll claim ∃m∈ℕ (u- )3 ≯2

 Notes prepared by sanjeev kumar shukla


 9971245238 or 8750558444
z

CONS. (u- )3 = u3 - 1/m3 - 3u2/m +3u/m2

> u3 - 1/m3 - 3u2/m+ 3u/m2

⇒ (u- )3>u3 – 1/m3 –(3u2/m)

Now, m3>m

⇒ 1/m3< 1/m

⇒ -1/m3> -1/m

⇒(u- )3> u3 – 1/m - (3u2/m)

=u3 – 1/m (1+3u2)

Now, (u- )3>2

If u3 - (1+3u2) >2

⇒ (1+3u2) < u3-2

⇒m > (1+3u2) / (u3-2)

Sunch an m exists :

By A.P. : m∈ℕ

(u- )3>2

Let s∈S (be any)

⇒s3<2 < (u- )3 (by (1))

⇒s<(u- ) ∀s∈S

 Notes prepared by sanjeev kumar shukla


 9971245238 or 8750558444
z

u - is an UB of S

⇒u≤u- ⇒ 0≤-1 ⇒0≥1 (which is not possible )

Sol 18 : Given: x<y

And u>0 ⇒ >0

⇒ <

By density theorem : ∃ r∈ℚ

<r< ⇒ x<ru<y

Theorem: Every convergent sequence is bounded

Proof:Let (xn) be cgt seq and let lim xn = x

Now for ε = 1 > 0 ∃m∈ℕ : |xn – x| < 1 ∀n∈ℕ

Now |xn| = |xn-x+x|

⇒|xn| ≤ |xn-x| + |x|

⇒|xn| ≤1+|x| ∀n≥m…………(1)

Let m=max { |x1| , |x2| , |x3| …….. |xn-1 , 1+|x| }

Then c;ear;y |xn| ≤m ∀n∈ℕ ⇒(xn) is bounded

As lim xn = z ⇒∃k1∈ℕ :|xn – z|<ε ∀n≥k1

 Notes prepared by sanjeev kumar shukla


 9971245238 or 8750558444
z

Theorem : (Sum Theorem0

Let X  x and Yy ( i.e lim xn = x , lim yn =y)

Then X+Y  x+y (i.e lim (xn+yn) = x+y)

Proof: Let ε>0 be given

(to find m∈ℕ L |(xn+yn) = (x+y)| <ε ∀n≥m)

Consider |(xn+yn) – (x+y) | = |(xn-x) + (yn-y)|

⇒|(xn+yn) – (x+y) | ≤ |(xn-x)| + |(yn-y)|…….(1)

Now lim xn = x ⇒ for > 0 ∃ m1∈ℕ : |xn-x| < ∀n≥m1 …..(2)

Also lim yn = y ⇒ for > 0 ∃ m2∈ℕ : |yn-y| < ∀n≥m2 …..(3)

Let m = max{m1,m2} then ∀n≥m, |xn-x|< ; |yn-y| < ……..(5)

Use (5) in(1) we get

|(xn+yn) =- (x+y) | < + ∀n≥m

⇒ |(xn+yn) – (x+y)| < ε ∀n≥m ⇒ lim (xn+yn) = x+y

 Notes prepared by sanjeev kumar shukla


 9971245238 or 8750558444
z

DIFFERENCE THEOREM

Let Xx , Yy then X-Y  x-y

(i.e if lim xn = x , lim yn = y, then lim(xn-yn=x-y)

Proof : Let ℇ> 0 be given,

(to find m∈ ℕ :|( xn - yn )-(X-Y)| <ℇ∀n≥m)

Consider |( xn - yn) – (x-y)|<ℇ∀n≥m

|( xn - yn) – (x-y)| = |( xn - x) – (yn-y)|

⇒ |( xn - yn) – (x-y)| ≤ |( xn - x)| + | (yn-y)|…………..(1)

Now proceed as in sum theorem

PRODUCT THOREM

Let Xx and Yy then XY  xy

(i.e. If lim xn=x and lim yn=y then lim xnyn=xy

Proof:

Let ε>0 be given (to find m∈ℕ : |xnyn – xy | < ε∀n≥m)

Consider |xnyn – xy| = |xnyn – xyn + xyn –xy |

⇒|xnyn – xy| = |(xn– x)yn + x(yn –y) |

 Notes prepared by sanjeev kumar shukla


 9971245238 or 8750558444
z

⇒|xnyn – xy| ≤ |yn| |xn – x| + |x| |yn – y| …….(1)

Now (yn) is cgt. ⇒ (yn) is bounded ⇒ M>0 : |yn| ≤M ∀


n∈ℕ……..(2)

Use (1) in (2)

⇒ |xnyn – xy| ≤M|xn – x| + |x| |yn – y|………..(3)

Let M* = max{M,|x|} then |xnyn – xy|≤M* ( |xn – x|+|yn – y |)


……..(4)

Now as lim xn = x ∴ for >0 ∃ m1∈ℕ : |xn-x|< ∀n≥m1

….....(5)

Also lim yn=y ∴ for >0 ∃ m2∈ ℕ : |yn-y|<

∀n≥m2……….(6)

Let m=max{m1,m2| then ∀n≥m we have

|xn-x| < , |yn-y| ≤ …………..(7)

Use (7) in (4) we obtain

|xnyn – xy|<M* ( + ) ∀n≥m

⇒|xnyn – xy| <ε ∀n≥m ⇒lim xnyn=xy

Lemma: Let lim yn=y and y≠0, then ∃k>0 and m 1∈ℕ:

|yn|>k ∀ n≥m1

 Notes prepared by sanjeev kumar shukla


 9971245238 or 8750558444
z

Proof: As y≠0 ⇒ |y|>0 choose 0<ε<|y| ………….(1)

Then, as lim yn=y ∴ for ε>0 ∃ m1∈ℕ:

|yn-y|<ε ∀ n≥m1……….(2)

Now |y|-|yn| ≤ |yn-y|<ε ∀n≥m1 (by (2))

⇒|y|-|yn| ≤ ε ∀n≥m1

⇒|y|-ε<|yn| ∀n≥m1

Let k=|y|-ε>0 ( ∵ 0<ε<|y| ⇒ ε<|y| ⇒ |y|=ε>0)

⇒|yn|>k ∀n>m1

QUOTIENT THEOREM

Let Xx and Yy with Y≠0 and y≠0,Then 

(i.e If lim xn=x ,lim yn=y,yn≠0 ∀n∈ℕ and y≠0

then lim( xn/yn)=x/y

Proof : We’ll show lim (1/yn) = 1/y , then by product


theorem

 Notes prepared by sanjeev kumar shukla


 9971245238 or 8750558444
z

lim( xn/yn) =lim(xn . 1/yn) = lim xn ∙ lim (1/yn) = x∙ =

Let ε>0 be given (to find m∈ ℕ : |1/yn – 1/y| ≤ε ∀n≥m)

Consider : |1/yn – 1/y| = | y-yn / yyn| = |1/y| ∙| 1/yn | ∙|yn-


y|…………(1)

By above lemma ∃k>0 and m 1∈ℕ : |yn|>k ∀n≥m1

⇒ 1/|yn| < ∀ n≥m1 , use this in (1) we’ll get

|1/yn – 1/y | ≤ |yn-y| ∀ n≥m1……….(2)

Let ε=εk|y|>0 as lim yn=y ∴ ∃m2∈ℕ :

|yn-y|< k|y|ε ∀n≥m2

Now let m=max{m1,m2}

Then ∀ n≥m we have

|1/yn – 1/y | < |yn-y| and |yn-y| < k|y|ε

⇒|1/yn – 1/y | < k|y|ε and ∀n≥m

⇒|1/yn – 1/y | <ε ∀n≥m ⇒ lim (1/yn) =

 Notes prepared by sanjeev kumar shukla


 9971245238 or 8750558444
z

SQUEEZE THEOREM

Let lim xn = l = lim yn and let xn≤zn<yn ∀n∈ℕ Then lim zn=l

Proof : Let ε>0 be given

As lim xn=l

∴ ∃m1∈ℕ : l-ε<xn<l+ε ∀n≥m1 ……………..(1)

Also lim yn=l ∴∃m2∈ℕ : l-ε<yn<l+ε ∀n≥m2 …………….(2)

Let m=max|m1,m2| then ∀n≥m we have

l-ε<xnl+ε

l-ε<ynl+ε

Now l-ε<xn<zn<yn< l+ε ∀n≥m

⇒ l-ε<zn< l+ε ∀n≥m ⇒ l;im zn=l

SQUARE THEOREMLet (xn) be sequence of non-negative terms subh


that lim xn=x then lim √xn = √x

Proof : Case 1 x=0 (T.S.T lim √xn = 0)

Let ε>0 be given (to find m∈ℕ : | √xn-0|<ε ∀≥m)

As lim xn =0 ∴ for ε2>0 ∃m∈ℕ : |xn-0|<ε2 ∀n≥m

⇒|xn|<ε2 ∀≥m

⇒xn<ε2 ⇒ √xn< ε ∀n≥m

⇒|√xn – 0| < ε ∀n≥m

 Notes prepared by sanjeev kumar shukla


 9971245238 or 8750558444
z

⇒ lim √xn =0

Case 2: If x≠0

Let ε>0 be given ( to fid m∈ ℕ : |√xn - √x | <ε ∀n≥m )

Consider |√xn - √x | = |(|√xn - √x |) (|√xn + √x |) /


(√xn + √x )

⇒|√xn - √x | = 1/(√xn + √x) ∙ |xn - x|………….(1)

Now √xn + √x > √x ⇒ 1/(√xn + √x) ≤ 1/√x………(2)

From (1) and (2) we have

|√xn - √x | ≤ |xn-x|…………(3)

Lim xn=x ∴ for √x ε>0 ∃m∈ℕ:

|√xn - √x| ≤ √x ε ∀n≥m


⇒ |√xn - √x| ≤ ε ∀n≥m ⇒lim √xn = √x

MODULUS THEOREM

Let lim xn =x then lim |xn| = |x|

Proof : Let ε>0 be given (to find m∈ ℕ : ||xn|-|x|| <ε ∀ n≥m

As lim xn=x

∴ ∃ m∈ℕ : |xn-x|<ε ∀n≥m………(1)

Now ||xn||x|| ≤ |xn-x| <ε ∀n≥m (by (1))

⇒ ||xn||x|| < ε ∀n≥m ⇒ lim |xn| = |x|

 Notes prepared by sanjeev kumar shukla


 9971245238 or 8750558444
z

Q4. Let x1=1, xn+1 = √ n∈ℕ

Show that (xn) is convergent find it’s limit

Soln: x1=1, x2=√ = 1.732

x3=√ = √ = √ = …….

We can see x1<x2<x3 ………

So we observe that (xn) is ↑ (increasing)

Now we’ll prove (xn) is ↑ i.e. xn≤xn+1 ∀n∈ℕ……….(1)

The result is true for n=1 ∵x2=1.732 > x1

Now let us assume that result is true for n=k

⇒xk≤xk+1…………(2)

We’ll show that result is true for n=k+1 (i.e. xk+1 ≤ xk+2)

As xk≤xk+1 by(2)

⇒2+xk ≤ 2+xk+1 ⇒ √ ≤ √

⇒xk+1 ≤ xk+2

∴By P.M.I xn ≤ xn+1 ∀n∈ℕ

⇒xn is ↑ ……………..(*)

Now we’ll show that (xn) is bounded above by 2

(i.e xn ≤ 2 ∀n∈ℕ)

For n=1, x1=1<2 ∴ result is true

 Notes prepared by sanjeev kumar shukla


 9971245238 or 8750558444
z

Assume that result is true for k

⇒xk≤2………(3)

We’ll show result is t rue for n=k+1 (i.e xk+1 ≤2)

As xk≤2 ⇒2+xk≤4

⇒√ ≤√ = 2

⇒√ ≤2 ⇒ xk+1 ≤2

∴ result is true ∀n∈ℕ By P.M.I

⇒xn ≤ 2 ∀n≤ℕ⇒ (xn) is bounded

By M.C.T (xn) is convergent

Let limit = x as xn+1 = √

⇒ lim xn+1 = √

⇒ x = √

⇒x2= 2+x ⇒ x2-x-2 = 0

⇒x2 – 2x+x-2 = 0

⇒(x-2) (x+1) = 0

⇒ x=2 , x=-1

But xn≥1 ∀n∈ℕ

⇒lim xn ≥1 ⇒x=-1 is not possible

⇒ x=2 ⇒ lim xn=2

 Notes prepared by sanjeev kumar shukla


 9971245238 or 8750558444
z

Q5: Let y1=√ , p>0, yn+1 = √ , n≤ℕ.

Show that (yn) is convergent . find its limit.

Solution:

y1=√ , y2 = √ = √ √ >√ = y1

⇒y1=√

We’ll show that (yn) is ↑ (i.e yn+1 ≥ yn ∀n∈ℕ)

For n=1 , as y2≥y1 ⇒ result is true

Assume that result is true for n=k

⇒yk+1 ≥ yk………….(1)

As yk+1≥ yk

⇒p+ yk+1 ≥ p+ yk ⇒ √ ≥ √ ⇒yk+2 ≥ yk+1

∴ result is true foe n+k+1 too and hence by P.M.I y n+1 ≥ yn


∀n∈ℕ

⇒(yn) is ↑

Now we’ll shoe (yn) is bounded

As yn+1 ≥ yn ⇒ √ ≥ yn

⇒ (p+yn) ≥ yn2

⇒yn2 – yn – p ≤0


⇒ (yn - ) (yn - ) < 0

 Notes prepared by sanjeev kumar shukla


 9971245238 or 8750558444
z

As yn ≥ y1 = √ ⇒ yn>√ >0

√ √
∴ yn ≮ ( ∴ < 0 )


⇒ yn< ∀ n∈ℕ⇒ (yn) is bounded ⇒(yn) is

convergent

Now let lim yn = y

As yn+1 = √

⇒ lim yn+1 = √

⇒ y= √ ⇒ y2-y-p =0


⇒ y =

As yn > 0 ∀n∈ℕ ⇒ lim yn ≥0 ⇒ y≥0

√ √
Now y = , y = < 0 ( ∵ √ > 1 ⇒ 1-√ <

0)

√ √
⇒ y = ∴lim yn =

Q6 : a+0 and z1>0 , Let zn+1 = √ ∀n∈ℕ

Show that (zn) converges, find its limit

Sol: zn+1 = √

⇒zn+12 = a+zn

⇒zn2 = a+zn-1

⇒(zn+1)2 - zn2 = (a+zn) - (a+ zn-1)

 Notes prepared by sanjeev kumar shukla


 9971245238 or 8750558444
z

⇒ (zn+1)2 - zn2 = zn – zn-1 ……………(1)

If zn>zn-1 ⇒ (zn+1)2 – zn2> 0 ⇒ zn+1> zn

And zn< zn-1 ⇒ zn+1< zn

i.e zn+1> zn >zn-1 ∀n∈ℕ and n>1

or zn+1< zn <zn-1 ∀n∈ℕ and n>1

⇒ (zn) is ↑ or (zn) is ↓ ⇒ zn is monotone

Now zn+1> zn or zn+1> zn

⇒√ > zn or √ < zn

⇒ a+zn> zn2 or a+zn< zn2

⇒zn2 – zn – a <0 or zn2 – zn – a > 0

√ √
⇒(zn - ) (zn - ) < 0 or its >0

√ √
But (zn - ) ≮ 0 (∵ <0 and zn> 0)

√ √
⇒ (zn - ) < 0 or (zn - ) > 0

√ √
⇒ zn< ∀ n∈ℕ or zn> ∀n∈ℕ


i.e. If zn+1> zn ∀ n∈ℕ then zn< ∀n∈ℕ

⇒ (zn) is bounded and ↑ sequence ⇒ (z n) is cgt


Now if zn+1< zn then zn> ∀n∈ℕ

⇒ (zn) is monotone and bounded ⇒ (z n) is cgt

 Notes prepared by sanjeev kumar shukla


 9971245238 or 8750558444
z

Let lim zn =z , as zn+1 = √

⇒ lim zn+1 = √

⇒z = √


⇒z2 = a+z ⇒ z2-z-a =0 ⇒z =

( ∵ zn ≥ 0 ∀n∈ℕ ⇒ lim zn ≥0 ⇒ z≥0 )

Q7 : x1 = a>0 , xn+1 = xn + for n∈ℕ;

Determine If (xn) converges or diverges.

Sol: Clearly xn>0 ∀ n∈ℕ ⇒xn+1 – xn = >0 ∀n∈ℕ

⇒ xn+1 – xn>0 ∀n∈ℕ ⇒xn+1>xn

⇒ (xn) is ↑, If (xn) is cgt then it has to bounded

⇒∃b>0 : xn≤b ∀n∈ℕ

Now (xn) is cgt ⇒ lim (xn+1 – xn ) =0………..(1)

Also in that case xn< b ⇒xn+1 – xn = ≥

⇒ lim (xn+1 - xn) ≥ >0………(2)

(1) and (2) are contrary to each other ⇒ (xn) can be


convergent (xn) is divergent

Q11: Let yn = + + ………………….+ ∀n∈ℕ

Sol: yn+1 – yn = + - >0 ∀n∈ℕ

⇒ yn+1 – yn> 0 ∀n∈ℕ ⇒(yn) is ↑…………(1)

 Notes prepared by sanjeev kumar shukla


 9971245238 or 8750558444
z

Also yn< + + …………+ ⇒yn< 1 ∀n∈ℕ⇒(yn) is

bounded…..(2)

By (1) and (2) and using M.C.T (y n) is bgt

Q12: xn = + + + …………………… + ∀n∈ℕ

Clearly xn+1 – xn = > 0 ∀n∈ℕ


)

⇒(xn) is ↑………..(1)

Now observe that

≤ ∀k≥2
)

⇒ ≤ - ∀k≥2
)

⇒xn ≤ 1+ (1- ) + ( - ) + ( - ) + …………….. +( - )

⇒xn ≤ 2- < 2 ∀n∈ℕ

⇒xn is bounded ∴ xn is cgt (by M.C.T)

Q13:

(a) lim ( ) = lim ( ( ) ))

= lim ( ) lim )

= e-1 = e

(b) lim ( ) = l i m (( ) )

= ( ( ) ) = e2

 Notes prepared by sanjeev kumar shukla


 9971245238 or 8750558444
z

( )
(c) lim ( ) = = = e
( )

(d) lim ( ) = (( ( )) )

= ( ( ( )) ) = e-1- =

Exapmle : Let a>0 , Let S1> 0 , and Sn+1 = (Sn+ ) for

n∈ℕ

Determine if (sn) is convergent

Soln: As Sn+1 = (Sn+ )

⇒ 2Sn+1 = (Sn+ )

⇒2Sn+1 Sn = Sn2 + a

⇒Sn2-2Sn+1Sn + a = 0

Now this eqn must have real roots

⇒ Discreminent ≥ 0

⇒4 S2n+1 – 4a > 0 ⇒ S2n+1 > a ⇒ Sn+1>√ ∀n∈ℕ

⇒Sn> min { √ , S1 } ∀n∈ℕ

⇒ (Sn) is bounded below……..(1)

Now we’ll show its decreasing

Sn – Sn+1 = Sn - (Sn + ) = ) ≥ 0 ∀≥2

⇒(sn) is decreasing……….(2)

 Notes prepared by sanjeev kumar shukla


 9971245238 or 8750558444
z

From (1) and (2) (Sn) is convergent (By M.C.T).

Let lim Sn =a

⇒S= ) ⇒S= + S

⇒ S= ⇒ S2 = a ⇒ s+ √

⇒ lim Sn = √

Definition : (SUBSEQUENCE)

Let X= (xn) be a sequence of real numbers and let n 1< n2<


………… be a strictly increasing sequence of natural
numbers then the seq. X 1= is called subsequence if (xn)

Examples Let (xn) = ( x1, x2, x2 , ………) then following are


some subsequences of (xn)

(1) (x1,x3,x5,x7,……..)
(2) (x2,x3,x4………)

More examples :

(1) X=(1, , , , …………..)

Then X1 = ( , , …………..)

(2) Let X = ( 1, , , , ……………….)

X1 = ( 1, , , , ……………….)

X2 = ( , , , ……………….)

 Notes prepared by sanjeev kumar shukla


 9971245238 or 8750558444
z

Theorem: If a sequence X=(xn) of real numbers converges to a real


number ‘x’ then any subsequence X1 = ( ) of X also converges to x.

Proof: Let ε>0 be given, to find m ℕ:

| | < ε ∀K≥m

As lim xn=x ⇒∃mℕ:

|xn-x|<ε ∀n≥m……….(1)

Now we know nk ≥ K ∀k∈ℕ

∴∀K≥m, nk ≥ m

⇒| | < ε ∀ K≥m

⇒ lim = x

Divergence criteria :

If a sequence X=(xn) of real numbers has either of the following


properties, then X is divergent

(i) X has two conver gent subsequences X’ = ( ) and


X’’=( ) whose limits are not equal
(ii) X is unbounded

Monotone Subsequence Theorem :

If X = (xn) is a sequence of reals then there is a


subsequence of X that is monotone

NOTE : PROOF IS OMMITED

 Notes prepared by sanjeev kumar shukla


 9971245238 or 8750558444
z

Q8: Given : (1) (an) is ↑ (bn) is ↓ sequence of real

(2) an ≤ bn ∀n∈ℕ

Show that: lim an ≤ lim bn and If lim an = η and lim bn = ξ


them an ≤ η ≤ ξ ≤bn

Sol: an ≤b1 ∀n∈ℕ

⇒(an) is ↑ and bounded above by b 1

⇒(an) is cgt. (By M.C.T)

Also lim (an) = Sup {an : n∈ℕ|

Let lim an = η……………(1)

Now (bn) ↓ also a1≤bn ∀n∈ℕ

⇒(bn) is monotone and bounded

⇒(bn) is convergent , and lim bn = Inf{bn L n∈ℕ}

Let lim bn = ξ……….(2)

We have to show : “η≤ξ”.

As η=Sup{an:n∈ℕ} ⇒ an≤η ∀n∈ℕ……….(3)

ξ = Inf{bn : n∈ℕ| ⇒ξ≤bn …………….(4)

Now an≤bn ∀n∈ℕ ⇒an-bn ≤ 0 ⇒lim (an-bn)≤0

⇒η=ξ≤0 ⇒η≤ξ ……………..(5)

Drom (3), (4), and (5) we get

 Notes prepared by sanjeev kumar shukla


 9971245238 or 8750558444
z

An ≤ η ≤ξ ≤bn ∀n∈ℕ

Q9: A is infinite sunset of ℝ, that is bounded above

Let u = Sup(A) (T.S.T ∃ (xn) with xn∈A and lim xn=u)

Sol: As Sup (A)=u ⇒u- is not an upper bound of A ∀ n∈ ℕ

⇒ ∀n∈ℕ∃ xn∈A : xn>u- ……………(1)

Also xn∈A ⇒xn≤u ∀n∈ℕ (∵Sup(A) = u)

∴u- ≤ xn ≤ u ∀n∈ℕ , by squeeze theorem lim x n = u

Blozano – weierstrass theorem

A bounded sequence of real numbers has a convergent


subsequence

Proof:By monotone subsequence theorem if (x n) is a


seq of reals then (xn) has a monotone subsequence
say X1 also (xn) is bounded

⇒X1 is bouonded

As X1 is monotone by M.C.T X 1 is convergent

 Notes prepared by sanjeev kumar shukla


 9971245238 or 8750558444
z

Theorem If X=xn is a sequence of real numbers then X  x iff each of


its subsequences converges to x.

Proof: if (xn)  x then we have proved every subsequence


of (xn) also converges to z.

Now let every subsequence of X converges to x then X


being a subseq of itself converges to x

Exresice 3.4

Q1: Let (xn) be a seq defined as

x2n-1 = 2n-1 and x2n =

⇒(xn) = (1, , 3 , , 5 , ………….) Clearly (xn) is

unbounded, bur the subseq X 1 = ( , , , ………….)  0

⇒ X1 is convergent

Q2: Show that if 0<c<1, then ) = 1

0<c<1, then xn+1 = ) > ) = xn

⇒ xn+1> xn ∀n∈ℕ⇒ (xn) is ↑

Also 0 < c < 1 ⇒ xn = ) < 1 ∀n∈ℕ

⇒ (xn) is bounded ∴ By M.C.T (x n) is convergent

Let lim xn = x ⇒lim x2n = x

 Notes prepared by sanjeev kumar shukla


 9971245238 or 8750558444
z

⇒ lim )= x

⇒lim √ = x

⇒√ = x

⇒√ = x ⇒ √ = x

⇒ x2 = x ⇒x=0 , x=1

Now xn ≥ x1 = c ⇒ lim xn ≥ c ≥ 0

⇒lim xn ≠ = ⇒ x≠0 ⇒ x=1

⇒ lim xn = 1 ⇒ lim ) = 1

Fibonacci sequence :

F: = (fn) is given by the Inductive definition

f1 = 1 , f2 =2 , fn+1 = fn-1 + fn ∈n≥2

Q3: Given (1) (fn) is a Fibonacci seq

(2) xn = and lim xn = L , find the value of L

Sol: By definition of (fn) ; xn ≥1 ⇒xn>0 or lim xn ≥ 1


⇒lim xn> 0

⇒ lim xn = 0

 Notes prepared by sanjeev kumar shukla


 9971245238 or 8750558444
z

Now xn = = = 1+

⇒xn = 1+ ⇒xn = 1+ ⇒ lim xn = 1+

⇒ L = 1 + ⇒L2-L-1 = 0

√ √
⇒L = as L>0 ⇒ L =

Q4: Show that following sequences are divergent

(a) ) (b) ( ))

Sol: (a) Let X’ = ( )= ) = ( ) )

⇒X’ = ( ) ⇒ lim = lim =0

Let X’’ = ( )= ) = ( ) )

⇒X’’ = ( )⇒ lim = lim = 2

∴ X has two convergent sequences with different limits


and hence by divergence criteria X diverges

(b) Let X’ = ( )= ( ) = (sin 4k × ) = (sin kπ ) = (0)

⇒ lim ( )= 0

Let X’’ = ( )= ( )) = (sin 4(k-3) × )

⇒ X’’ = (sin (2kπ - )) = (-sin ) = (1)

⇒ lim ( )= 1

 Notes prepared by sanjeev kumar shukla


 9971245238 or 8750558444
z

∴ X has two convergent sequences with different


limits and hence by divergence criteria X diverges

Q5: first Let lim zn = z then (xn) and yn) being subsequences of
(zn) also converge to z

Now conversely assume lim xn = lim yn = z

We’ll show lim zn =z, Let ε>0 be given

As lim xn = z ⇒∃k1∈ℕ :|xn – z|<ε ∀n≥k1

And lim yn = z ⇒∃k2∈ℕ :|yn – z|<ε ∀n≥k2

(1) ⇒|z2n-1 –z| <ε ∀2n-1≥2k1-1


(2) ⇒|z2n=z|<ε ∀2n≥2k2

Let k=max{2k1=1, 2k2| then ∀n≥k, |n=z|<ε

⇒lim zn=z

Chapter 3.2 Theorems

Bounded sequence :

Let (xn) be a sequence of real numbers then (x n) is said to


be bounded if ∃ M>0 such that |x n| ≤M ∀n∈ℕ

Examples:

)
(1) Let (xn) = ( ) = (1 , , , , , ……….)

Then clearly |xn| = ≤ 1 ⇒ |xn| ≤1 ∀n∈ℕ⇒ (xn) is

bounded

 Notes prepared by sanjeev kumar shukla


 9971245238 or 8750558444
z

(2) Let (xn) = (1 , 2 , 1 , -1 , 1 , -2 , 1 , 2 , 1 , 2……….)


Then |xn| ≤ 2 ∀n∈ℕ⇒ (xn) is bounded
(3) Let (xn) = (1 , -1 , 1 , -1, 1 , ………….)
|xn| = 1≤1 ∀n∈ℕ⇒ |xn| ≤ 1 ∀n∈ℕ⇒ (xn) is bdd

Q5: Let xn= ) , n∈ℕ show that

(a) Xn+1< xn ⟺ ( ) < n

(b) Show that (xn) is ultimately decreasing and bounded


below so that x=lim xn exists use the fact that the
subsequence x2n also converges to x , to conclude
that x=1

Sol: (a) xn+1< xn ⟺ ( ) < n (we’ll show )

⟺ ) < )

⟺(n+1)n< nn+1

⟺(n+1)n < n2 ∙ n

⟺(n+1)n < n

⟺( ) < n

∴ xn+1< xn ⟺ ( ) < n……..(1)

Now observe

) )
( ) = 1+n ∙ + +…………..+

 Notes prepared by sanjeev kumar shukla


 9971245238 or 8750558444
z

= 2 + ( )+ ( )( )+ … … … . + ( ) ( )… … . .

< 2 + + + ………. +

< 2 + + + …….. + (∵ nn-1 ≤ n! )

( )
⇒ ( ) < 2 + ⇒ ( ) < 3 - < 3 ≤ n n≥3

⇒ ( ) < n n≥3

⇒ xn+1< xn ∀≥3 (By (1))

⇒(xn) is ultimately decreasing also x n ≥ 1 ⇒ (xn) cgt

Let lim xn = x

(c) lim x2n = x

⇒lim( ) )= x

⇒ ( ) ) = x

⇒ lim ( ) lim ( ) = x2

⇒lim xn = x2 ⇒ x=0 , x=1

But xn≥1 ⇒ lim xn ≥ 1 ⇒ x≥1 ⇒x=1

Q7: (a) Let xn = ( ) then lim xn

Also = .( ) / is a subsequence of (xn)

 Notes prepared by sanjeev kumar shukla


 9971245238 or 8750558444
z

∴ lim = lim xn = e

(b) ( ) = (( ) ) …………..(1)

Now let xn = ( ) = lim xn = e

Also x2n = ( ) is a subseq of xn

⇒ lim x2n = lim xn = e

⇒lim ( ) = e

∴ By (1) lim ( ) = ( ( ) ) =

(c)( ) = .( ) / ……………(1)

N o w a s p r o v e d i n p a r t ( a ) l i m .( ) / = 2

∴ B y ( 1 ) l i m .( ) / = . ( ) / = e2

(d) ( ) = ( ) = ( ) = ( ) ( )

( )
⇒ lim ( ) = lim ( ( ) ) = = e2
( )

 Notes prepared by sanjeev kumar shukla


 9971245238 or 8750558444
z

Q8: Determine the limits of following

(a) ( ) ) ( b ) (( ) )

Now let xn = ) ⇒ lim xn = 1

Then x3n = ) is a subseq of xn

⇒ lim x3n= 1 ⇒ lim ) = 1

∴ By (1) ) = ( ) ) = = 1

(b) (( )) = (( ) ) = (write details )

Q9: Given: Every subseq of (xn) has a subseq that converges to


zeroq

Show that (xn) converges to zero

Sol: let if possible (xn) does not converge to 0

⇒∃ ε0 : ∀m∈ℕ we can find k∈ℕ : k>m

And |xn-0| ≮ε0

⇒∀m∈ℕ∃ k∈ℕ : k>m

And |xn| ≥ ε0

∴for m1∈ℕ∃ n1∈ℕ : n1>m1

And | | ≥ε0

⇒ for m1>n1 and m2∈ℕ ∃ n2∈ℕ : n2>m2>n1

And | | ≥ε0

 Notes prepared by sanjeev kumar shukla


 9971245238 or 8750558444
z

Continuing like this we get a subseq convergind to zero

(∵ if ( ) has a subseq say( ) converging to zero then for ε 0


∃ m∈ℕ : | <ε0 for all n’k’> m which is contradiction to (1)

Which contradicts the fact that every subseq o f (xn) has a


subseq converging to zero

∴ lim xn ≠ 0 is not possible ⇒ lim xn =0

⇒ (xn) converges to zero

Q11: Given (1) xn≥0 ∀n∈ℕ

(2) Lim ) ) exists

Show that lim xn exists (i.e (xn) is cgt)

Sol: Let Lim ) ) = x (as the given limit exists)

Now, xn = |xn| = ) (∵ xn ≥0)

⇒ lim xn = |lim ) | = |x|

⇒ lim xn = |x| ⇒(xn) is cgt

Now ) is cgt

⇒lim ) = lim )

⇒lim x2k = - lim x2k-1

⇒ |x| = -|x| (∵ lim xn = |x| and (x2k) , (x2k-1) are subseq )

⇒ 2|x| = 0 ⇒ |x| = 0 ⇒x=0

⇒ Lim ) )= 0

And also lim xn = 0

 Notes prepared by sanjeev kumar shukla


 9971245238 or 8750558444
z

Q12 : Show that if (xn) is unbounded then there exist a

subseq ( ) such that lim ( )= 0

Sol: as (xn) is unbdd ⇒ for 1∈ℕ, ∃n1 ∈ℕ :

>1

⇒ for 2∈ℕ, ∃n2>n1 : > 2

Continuing like this for nk> nk-1

1 >k

⇒0 ≤ < ⇒ lim = 0 ⇒ lim = 0

Q14: Given: (1) (xn) is bounded seq

(2) S=Sup{ xn : n∈ℕ } . Show that if S ∉{xn:n∈ℕ}

To show that : there is a subseq of (xn) that


converges to S

Sol: As S= Sup{xn:n∈ℕ}

For 1∈ℕ, ∃n1 ∈ℕ: > S-1

For 2∈ℕ, ∃n2>n1: > S-

Continuing like this we see ∀k∈ ℕ

∃nk> nk-1 :

> S- ………….(1)

Also ∀k∈ℕ, ≤ S (∵ Sup{xn : n∈ℕ } = S )……….(2)

 Notes prepared by sanjeev kumar shukla


 9971245238 or 8750558444
z

From (1) and (2) S- < ≤ S

As lim (s- ) = S , lim S=S

∴By squeeze theorem lim =S

Definition (Cauchy sequence)

A seq X= (xn) of real numbers is said to be a Cauchy seq if for


every ε>0 ∃H(ε) ∈ℕ: |xn-xm| <ε ∀n,m ≥ H(ε)

Example: (a) (xn) = ( ) is a Cauchy seq

Let ε>0 be given1

Now |xn-xm| = | - | ≤ + …………(1)

Choose H(ε) ∈ℕ : H(ε) > , then ∀n,m ≥ H(ε)

n> , m> ⇒ < , <

By (1) |xn – xm| < + = ε ⇒ |xn – xm| < ε

⇒ (xn) is Cauchy sequence

 Notes prepared by sanjeev kumar shukla


 9971245238 or 8750558444
z

Lemma: (convergent  cauchy)

If X=(xn) is convergent seq of reals then X is a cauchy


sequence

Proof: Let lim xn=x

Let ε>0 (to find H(ε) ≤ : |xn – xm| <ε ∀n,m ≥ H(ε))

As lim xn = x ⇒for > 0 ∃H(ε)∈ℕ :

|xn-xm| < ∀n≥H(ε) ………….(1)

Now |xn-xm| = | (xn –x) – (xm – x) |

⇒|xn-xm| ≤ |xn –x| + |xm –x| …………..(2)

Now by (1) for n,m≥H(ε)

|xn –x| <

Use this in (2) we get,

|xn –xm| < ε ∀n,m≥H(ε)

⇒(xn) is Cauchy

 Notes prepared by sanjeev kumar shukla


 9971245238 or 8750558444
z

Lemma: (Cauchy  bounded)

A cauchy sequence is bounded

Proof: Let X = (xn) be Cauchy seq

∴ for ε = 1 ∃H∈ℕ:

|xn-xm| < 1 ∀n,m ≥ H

For m=H, |xn – xH | <1 ∀n≥H……(1)

Now |xn| = |xn – xH + xH|

≤ |xn – xH | + |xH|

< 1 + |xH| ∀n≥H (by (1))

⇒ |xn| < 1+ |xH| ∀ n≥H ……….(2)

Let M=max{ |x1| , |x2| ………. |xH-1| , |xH|+1}

Then |xn| ≤ M ∀n∈ℕ ⇒ (xn) is bounded

Cauchy convergence criteria (C.C.C)

A sequence of reals is convergent iff it is Cauchy

Proof: We know by a lemma that every convergent seq is


Cauchy. Conversely let (x n) be a Cauchy seq

We’ll show (xn) is convergebt

(xn) being Cauchy is bounded

 Notes prepared by sanjeev kumar shukla


 9971245238 or 8750558444
z

∴ By Bolzano-weierstrass theorem (xn) has a convergent


subsequence sau ( ) and that lim = x*

We’ll show lim xn = x*

Let ε>0:

As (xn) is Cauchy ⇒∃H∈ℕ:

|xn-xm|< ∀n,m≥h1………….(1)

Also lim = x* ∴∃H2∈ℕ : H2≥H1(say)

In the set |n1,n2,………….|, with | –x*|<

Now by (1) as H2≥H1

∴|xn- | < ∀n≥H1 ……………..(2)

Now |xn –x*| = |xn - + – x*|

≤|xn - | + | – x*|

≤ ε ∀n≥H1 (by (1) and (2) )

⇒ |xn –x*| <ε ∀n≥H1

⇒ lim xn =x ⇒ (xn) is convergent

Exercise 3.5

Q1: Give an example of a bounded sequence that is not cgt.

 Notes prepared by sanjeev kumar shukla


 9971245238 or 8750558444
z

Q2: Show that following sequences are Cauchy directly from


the definition

(a) ( ) (b) ( )

Q3: Show deirectly from the definition that following are not
Cauchy sequences

)
(a) ) ) (b) ( ) (c) (ln n)

Sol 1: Let xn = (-1)n

Then |xn| < 2 ∀n∈ℕ ⇒(xn)is bounded

But (xn) is Cauchy as we’ll see below ∀H∈ ℕ

Let m=2H, then m>H and let n=2H -1>H

Now |xn –xm| = |x2H-1 –x2H | = |-1-1| = 2

So far ε = 1>0

|xn-xm| = 2>€ε ∀H∈ℕ

⇒(xn) is not Cauchy

Sol 2:

(a) Let xn = = 1 + and ε>0 (be given)

The |xn – xm| = | (1+ ) – (1+ ) |

⇒ |xn – xm| = | – 1 | ≤ +

⇒|xn – xm| < + ………(1)

 Notes prepared by sanjeev kumar shukla


 9971245238 or 8750558444
z

(Now to find H∈ℕ : n,m>H) ⇒|xn –xm|<ε )

As n>H , m>H ⇒ < , <

From (1) |xn –xm| < , ∴|xn –xm|<ε if < ε

∴ > ⇒H> , chose H∈ℕ : H>

Then ∀n,m>H, |xn-xm|<ε ⇒(xn) is Cauchy

(b) Let xn = 1+ + ……..+

Let for m>n


|xn –xm | = )
+ )
+………..+

⇒|xn –xm | = )
+ )
+………..+

⇒|xn –xm |≤ + + + ……….

⇒|xn –xm |≤ (1+ + + ……….

⇒|xn –xm |≤ . /

⇒|xn –xm |≤ ( )<

⇒|xn –xm |< ∴|xn –xm |<ε, if <ε

⇒2n-1>

⇒(n-1) > ln( )

∴ |xn –xm |<ε if n> ln( ) +1

So chose H∈ℕ : H> ln( ) +1

Then ∀m>n≥H , n> ln( )

 Notes prepared by sanjeev kumar shukla


 9971245238 or 8750558444
z

And hence |xn –xm | < ε ∀n,m≥H

⇒(xn) is acuchy seq

Sol 3:

(a) See solution (1)


)
(b) Let xn = n+

For any h∈ℕ, if n=2H, m=2H-1


Then |xn –xm| = |x2H –x2H-1|
) )
= | ) |

⇒|xn –xm| = | | = | |

⇒|xn –xm| = > 1

∴ for ε = 1 , ∄ any h∈ℕ : |xn –xm| < 1 ∀n,m≥H


⇒(xn) is not Cauchy
(c) Let xn = ln(n)
Then for any h∈ℕ
If n=4H and m=2H
|xn –xm|= |ln4H – ln2H|
⇒|xn –xm| = (ln2) ≮ε
If ε=ln2 +1 >0
∴(xn) is not Cauchy

 Notes prepared by sanjeev kumar shukla


 9971245238 or 8750558444
z

Q4: Given (xn) and (yn) are not Cauchy seq show that

(a) (xn + yn) is Cauchy sequence


(b) (xn yn) is Cauchy sequence

Sol:
(a) Let ε>0 be given (to find H∈ ℕ : ∀n,m>H)

Consider |(xn+ym) – (xm+ym)| ≤ |xn –xm| + |yn –ym| …….(1)

Now (xn) is cauchy ∴∃H1∈ℕ:

|xn -xm| < ∀n,m≥H1 …………..(2)

and (yn) is cauchy ∴∃H2∈ℕ:

|yn -ym| < ε ∀n,m≥H2 …………..(3)

Choose H=max{H1 , H2}

Then |xn –xm| < and |yn –ym| < ∀n,m≤H

Use this in (1) we get

|(xn + yn) – (xm + ym ) |<ε ∀n,m≥H

⇒((xn + yn)) is Cauchy seq

(b) Let ε>0 be given (to find H∈ ℕ : ∀n,m>H)

Consider |xnyn – xmym| = |xnyn – xmyn + xmyn – xmym |

⇒|xnyn – xmym| ≤ |xn – xn| |yn| + |xn| |yn-ym| …….(1)

Now (xn) and (yn) being cauchy are bdd ∴∃M>0 :

|xn|≤M . |yn|≤M ∀n∈ℕ


 Notes prepared by sanjeev kumar shukla
 9971245238 or 8750558444
z

Use this in (1) we obtain

|xnyn-xmym|≤M (|xn-xm| + |yn-ym|) ………..(2)

Now (xn) is Cauchy ∴∃H1∈ℕ :

|xn –xm|≤ ∀n,m≥H1 …………(3)

and (yn) is Cauchy ∴∃H2∈ℕ :

|yn –ym|≤ ∀n,m≥H1 …………(4)

Choose H=max{H1 , H2}

Then |xn –xm| < , |yn –ym| < ∀n,m≥H…………(5)

From (2) and (5) we get

|xnyn – xmym| < ε ∀n,m≤H

⇒(xn yn) is cauchy

Q5: If xn=√ , show that (xn) satisfy lim |xn+1-xn| =0 , but is not
Cauchy

Q6: Let p be a given natural number Given example of a sequence


(xn) that is not Cauchy sequence but that satisfies lim|x n+p – xn|
=0

Q7: Let (xn) be a Cauchy sequence such that x n us an integer, ∀n∈ℕ


show that (xn) is ultimately constant.

 Notes prepared by sanjeev kumar shukla


 9971245238 or 8750558444
z

Sol 5: | x n + 1 – x n | = |√ √ | = √ √

⇒lim|xn+1 –xn| = lim = 0


√ √

Now we’ll show (xn) is not cauchy

Let H∈ℕ (be any) then for n=4H, m=H

|xn-xm| = |√ √ √ ≥ 1 ⇒|xn –xm| ≥1

∴ |xn – xm| ≮ε, if ε = > 0, for any H∈ℕ

⇒(xn) is not Cauchy

Sol 6: Let xn = 1 + + + ……………….. +

then (xn) is not Cauchy , for n>m

|xn – xm| =

|( )

( )|

⇒|xn – xm|= | |

⇒|xn – xm|=

Let H∈, be any then for n.2H, m=H

|xn – xm| =

⇒|xn – xm| ≥ ⇒|xn –xm| >

∴|xn-xm| ≮ ε, for ε = , ∀H∈ℕ

 Notes prepared by sanjeev kumar shukla


 9971245238 or 8750558444
z

∴(xn) is not Cauchy

While xn+p = 1+ + + ………….+

∴ xn+p = 1+ + + ……….. + + + ……..….+

⇒ xn+p – xn = + + ………….+

⇒ 0 < |xn+p –xn| <

⇒lim |xn+p –xn| =0

Sol 7: As (xn) is cauchy

∴for 1>0 ∃H∈ℕ ; |xn –xm|<1 ∀n,m≥H

Now xn and xm are integers

∴|xn –xm| <1 ⇒xn = xm ∀n,m≥H

(xn) is ultimately constant

Q8: Show directly that a bounded monotone increasing


sequence is Cauchy

Q9: Of 0<r<1 and |xn+1 – xn|<rn ∀n∈ℕ. Show that (xn) is a


Cauchy sequence

 Notes prepared by sanjeev kumar shukla


 9971245238 or 8750558444
z

Sol 8: Let (xn) be bounded and increasing as (x n) is


bounded as (xn) is bounded

⇒{xn : n∈ℕ} is bounded

⇒Sup{xnLn∈ℕ} exists (By completeness property_

Let u=Sup{xnLn∈ℕ}

Claim: (xn) is Cauchy

Now let ε>0 be given (to find H∈ ℕ : |xn –xm| <ε ∀n,m≥H)

As u=Sup{xn:n∈ℕ}

⇒ ∃H∈ℕ: u- <hH< u …………(1)

Now ∀ n≥m≥H as (xn) is ↑

∴ u≥xn≥xm≥xH> u-

Also u <u+

∴u+ > xn ≥ xm> u - ∀n≥m≥H

⇒ |xn –xm| <ε ∀n≥m>H

⇒(xn) is Cauchy

Sol 9: for m>n

|xn –xm| = |xn –xn+1 + xn+1 – xn+2 + ………+ xm-1 – xm|

⇒|xn –xm| ≤ |xn –xn+1| +| xn+1 – xn+2 | + ………+ |xm-1 – xm|

⇒|xn –xm| < rn + rn+1 + ………..+ rm-1

)
⇒|xn –xm| ≤ < ∀m>n ……………….(1)
) )

 Notes prepared by sanjeev kumar shukla


 9971245238 or 8750558444
z

⇒|xn –xm| < ∀m>n

Let ε>0 be given (to find H∈ ℕ : ∀m>n≥H |xn-x|<ε

Now 0<r<1 ⇒lim rn =0

∴ for (1-r) ε>0 ∃H∈ℕ:

|rn-0|<(1-r)ε ∀n≥H

⇒|rn|<(1-r)ε ∀n≥H

⇒rn< (1-r) ε ∀n≥H ………(2)

From (1) and (2)

|xn –xm| <ε ∀m>n≥H

⇒(xn) is Cauchy

Q10: if x1≤x2 are arbitrary real numbers and x n = ) for

n>2 show that (xn) is cgt. Find limit

Q11: If y1<y2 are arbitrary real numbers and yn = +

For n>2, show that (yn) is cgt. find limit

Sol 10: as x2>x1 ⇒ x2 –x1>0, let L = x2-x1

we’ll show |xn+1 – xn| = L ( )

for n=1 |x2 – x1| = L

∴result is true

 Notes prepared by sanjeev kumar shukla


 9971245238 or 8750558444
z

For n+2 |x3 – x2 | = | ( x1 + x2 ) – x2 |

= |x2 – x1 | = =

∴result is true for n=2

Assume result is true for n=k

|xk+1 – xk | = ………….……(1)

Now |xk+2 – xk+1 | = | (xk + xk+1 ) – xk+1 |

= | xk+1 – xk | = =

∴result is true for n=k+1

∴|xn+1 – xn| =

As in question 9 for m>n

|xn-xm| ≤ + + + ……………. +

⇒|xn – xm| ≤ ( 1+ + +………….. +

⇒|xn – xm| < , let ε >0 be given ( to find H∈ ℕ |xn – xm|

< ε ; ∀m>n)

Now lim = 0 ∴for > 0 ∃H∈ℕ:

< ∀n≥H …………….(2)

Now from (1) and (2)

|xn – xm| < ε ∀m>n≥H ⇒ (xn) is Cauchy and hence is cgt

 Notes prepared by sanjeev kumar shukla


 9971245238 or 8750558444
z

We’ll show that

X2n+1 = x1 + + + + ………… +

we see x3 = (x1 + x2 ) = x1 + x2 = x1 + (L + x1 )

⇒x3 = x1 +

x5 = (x3 + x4 ) = x3 + ( x2 + x3 )

⇒x5 = x3 + x2 = x3 + (L+x1)

⇒x5 = (x1 + ) + x1 + L

)
⇒x5 = x1 L = x1 +

⇒x5 = x1 + + e.t.c ………….

∴x2n+1 = x1 + + + ……… +

As (xn) is cgt ⇒ lim xn = lim x2n+1

⇒ lim xn = x1 + ( 1+ + + ……………. +

Lim xn = x1 + . / = x1 + L = + x2

Sol 11: |yn – yn+1| = |yn - (yn + yn-1 |

⇒|yn – yn+1|= | yn - yn-1|

⇒|yn – yn+1| = |yn - yn-1|

 Notes prepared by sanjeev kumar shukla


 9971245238 or 8750558444
z

⇒|yn – yn+1| = ∙ |yn-1 - yn-2|

⇒|yn – yn+1| =( ) |yn-1 - yn-2|

⇒|yn – yn+1| =( ) |yn-2 - yn-3|

⇒|yn – yn+1| =( ) |yn-2 - yn-3|

……………………………………………………..
……………………………………………………..
……………………………………………………..

⇒|yn – yn+1| =( ) |y2 - y3|

⇒|yn – yn+1| =( ) (y2 - y3) (∵y2> y1 )

Now for m>n , as in Question 9

⇒|yn – ym| =|yn – yn+1 + yn+1 – yn+2 ………… + ym-1 – ym |

≤|yn – yn+1 |+ |yn+1 – yn+2|………… + |ym-1 – ym |

⇒ | y n – y m | ≤ ( y 2 – y 1 ) [( ) () () ]

( )
⇒|yn – ym| ≤ (y2 – y1) ( ) 0 1

⇒|yn – ym| ≤ 3(y2 – y1) ( ) ∀m>n …………..(1)

Now lim ( ) = 0 ∴ ∀ε>0 ∃ H∈ℕ :

( ) < ∀n≥H …………….(2)


)

From (1) and (2) we get |yn –ym|<ε ∀m>n≥H

 Notes prepared by sanjeev kumar shukla


 9971245238 or 8750558444
z

⇒ (yn) is Cauchy and hence (yn) is cgt

Find limit yourself (Ans. y1 + y2)

Definition

Let (Sn) be a seq in ℝ . we define

limSupSn =

limInfSn =

Theorem :Let (Sn) be a sequence in ℝ

(1) If limSn is defined ( in ℝ∪{±∞}), then


limInf = limSn = limSupSn
(2) If limInfSn = limSupSn, then limSn is defined and
limSn = limSupSn = limInfSn

Proof: (i) Suppose limSn = ±∞.

Let Un = Inf{Sn:n>N} and Vn =Sup{Sn:n>N} then

limSupSn = and limInfSn = for any m>0, as


limSn = +∞

∴∃N∈ℕ:

Sn>m ∀n>N

⇒Vn≥m…………..(1)

Now ∀m>N we have

 Notes prepared by sanjeev kumar shukla


 9971245238 or 8750558444
z

{Sn:n>m} ⊆ {Sn:n>N}

⇒ Inf{Sn:n>m} ≥Inf{Sn:n>N} ∀n>N

⇒Um≥Un≥m ∀m>N

⇒Um≥m ∀m>N ⇒ = +∞

⇒limInfSn = +∞

Similarly we can show liimSupS n = +∞

Also when limSn = -∞ case is similar

Now Let limSn=S, S∈ℝ

Let ε>0 be gien then ∃N∈ℕ:

S-ε<Sn<S+ε ∀n≥N

Now Vn = Sup{Sn:n>N} ≤ S+ε ∀ε>0

⇒Vn≤S+ε ∀ε>0

Also if m>N, then

{Sn:n>m} ⊆ {Sn:n>N}

⇒ Sup{Sn:n>m} ⊆ Sup{Sn:n>N}

⇒ Vm ≤ Vn ≤ S+ε ∀ε>0, ∀m>N

⇒ Vm≤S+ε ∀m>N

⇒ lim Vm≤S+ε ∀ε>0

⇒ limSupSn ≤S+ε ∀ε>0

⇒ limSupSn≤S ……………. *⃝1

 Notes prepared by sanjeev kumar shukla


 9971245238 or 8750558444
z

Also S-ε<Sn ∀n>N

⇒S-ε≤Un where Un={Sn:n>N}

⇒S-ε≤Un≤Um ∀m>N

⇒S-ε≤ ⇒S-ε ≤ limInfSn ∀ε>0

⇒S≤limInfSn ……….*⃝2

Now we know

limInfSn ≤ limSupSn

*⃝1 and *⃝2 ⇒S≤limInfSn ≤ limSupSn ≤ S

⇒limInfSn = limSupSn = S

(iii) (A) Let limSupSn = + ∞ = limInfSn


T.S.T limSn = +∞
Let M be any +ve real no.
As limInfSn = +∞
⇒ = ∞ ,Un = Inf{Sn:n>N}
∵ = ∞ ∴∃k∈ℕ:
Un>M ∀N>k
Or in particular Uk>M
⇒ Inf{Sn:n>k}>M
⇒Sn>m ∀n>ks
⇒limSn = +∞

(B) If limSupSn = -∞ = limInfSn


T.S.T limSn = - ∞
Let M∈ℝ be any real o
As limSupSn = -∞

 Notes prepared by sanjeev kumar shukla


 9971245238 or 8750558444
z

⇒limVn = -∞
⇒∃k∈ℕ:
Vn<M ∀N≥k
In particular Vk<M …………….(1)

Now ⇒ Sup{Sn:n>k} <M

⇒Sn<M ∀n>k

⇒limSn = -∞

(C) If limSupSn = limInfSn = S, S∈ℝ

Let ε>0 be any no. (to find k∈ℕ : |Sn-S| <ε ∀n≥k)

As limSn = S

⇒ = S, Vn = Sup{Sn:n>N}

⇒∃k1 ∈ ℕ:

|vn – S|<ε ∀N≥k1

⇒s-ε < Vn< s+ε ∀N≥k1

⇒ Vn< s+ε ∀N≥k1

In particular <s+ε

⇒Sip{Sn:n>k1} < s+ε

⇒ Sn< s+ε ∀n≥k1 ……………(1)

Also limInfSn = S

⇒limUn = s, Un=Inf{Sn:n>N}

 Notes prepared by sanjeev kumar shukla


 9971245238 or 8750558444
z

⇒∃k’’∈ℕ:

s-ε<Un<s+ε ∀N≥k’’

⇒ s-ε < Un ∀ n≥k’’

In particular for N=k’’

s-ε <

⇒s-ε<Inf{Sn:n>k’’}

⇒s-ε < Sn ∀n≥k’’ …………..(2)

Let k=max{k’ , k’’}, then

s-ε<Sn<s+ε ∀n≥k

⇒|Sn – S|<ε ∀n≥k

⇒limSn = S

Example: Let (Sn) = ) ) find limSupSn and limInfSn

Sol: Un = Inf{Sn:n>N} = Inf{1 ) :n>N}

⇒Un = Inf{

⇒Un = 0 ⇒limUn = 0 ⇒ limInfSn = 0

Similarly check limSupSn = 2

Example: Let xn = ) ( ) n∈ℕ, find limSupxn and liminfxn

Sol: Vn = Sup{xn:n>N}

=Sup{xN+1 , xN+2 , xN+3,……..}

 Notes prepared by sanjeev kumar shukla


 9971245238 or 8750558444
z

=Sup { ) ( ) ) ( ) , ………….}

={

limVn = 1 ⇒limSupxn = 1

Un = Inf{ ) ( ), ) ( ) …………}

( )
={
( )

⇒limUn = -1 ⇒limInfxn = -1

Definition:

Let (Sn) be a seq in ℝ. A susequential limit is any real


numbers or symbol + ∞ or – ∞ that is the limit of some
sunsequence od (Sn)

Theorem:

Let (Sn) be any sequence in ℝ, and let S denote the set of


subequential limits of (Sn)

(i) S is non empty


(ii) Sup S = limSupSn , Inf S = limInfSn

Proof: Not required (Not in the syllabus)

 Notes prepared by sanjeev kumar shukla


 9971245238 or 8750558444
z

Infinite series :

An infinite series is an expression of the form


∑ = am + am+1 , am+2 , ………

Note:We’ll often take m=0, or m=1 in this set of notes we’ll


assume m=1. ∴for us infinite seriws is ∑ ,We’ll ignore
n=1,2,………∞, and we’ll write an infinite series as ∑

Sequence od partial sums :

Let ∑ be an infinite series then a sequence (S n) defined as

Sn = a1 + a2 + a3 + ……… + an ,is called sequence of partial sums of ∑

Definition:

An infinite series is convergent iff (S n) is cgt

An infinite series is divergent iff (S n) is dgt

Series of non-negative terms :

A series ∑ is of non-negative terms if sn≥0 ∀ n∈ℕ

Note :S.O.P.S of series of non-negative terms is always ↑ seq

Let ∑ be series of non-negative terms and (Sn) be S.O.P.S

Then Sn+1 – Sn = an+1 ≥0

⇒Sn+1 – Sn ≥ 0 ⇒Sn+1 ≥ Sn ∀n∈ℕ

 Notes prepared by sanjeev kumar shukla


 9971245238 or 8750558444
z

⇒ (Sn) is ↑

As series ∑ is cgt ⟺(Sn) is cgt and (Sn) is ↑

∴ (Sn) is cgt ⟺ (Sn) is bdd

∴∑ is cgt ⟺ (Sn) is bdd above

If |r| ≥ 1 ⇒|rn| ≥ 1 ∀n∈ℕ

⇒lim|rn|≥1

⇒ lim|rn|≠0

Also Of a≠0

lim arn = alim(rn) ≠ 0 (∵ lim|rn| ≠ 0 ⇒ limrn ≠ 0)

∑ is no convergent

p-series test :

The series ∑ is cgt ⟺ p >1

is dgt ⟺ p≤1

 Notes prepared by sanjeev kumar shukla


 9971245238 or 8750558444
z

Cauchy convergence criteria:

∑ is convergent ⟺ ∀ ε>0 ∃ N∈ℕ : ∑ < ε ∀ n≥m>N

Proof: Let (Sn) be S.O.P.S of ∑

Then ∑ is cgt ⟺ (Sn) is cauchy

⟺ ∀ ε>0 ∃ N∈N:

|Sn – Sm| <ε ∀ n≥m>N

⟺ ∑ ∑ < ε ∀ n≥m>N

⟺ ∑ < ε ∀ n≥m>N

⟺| am+1 + am+2 + …… + an |<ε ∀ n≥m>N

Comparision test:

Let ∑ be a series where an ≥ 0 ∀ n∈ℕ

(i) If ∑ converges and |bn| ≤ an for all


n∈ℕ then ∑ converges
(ii) If ∑ = + ∞ and bn ≥ an ∀n∈ℕ, then
∑ =+∞

 Notes prepared by sanjeev kumar shukla


 9971245238 or 8750558444
z

Proof: (i) Let ε>0 n given

Now for n≥m

∑ ≤ ∑ ≤∑ ……………(1)

Now ∑ is cgt ∴∃N∈ℕ:

∑ < ε ∀n≥m>N

As an ≥ 0 ∀n∈ℕ

⇒ ∑ < ε ∀n≥m>N ……………..(2)

From (1) and (2)

∑ < ε ∀ n≥m>N

⇒ ∑ is comvergent

(ii) Let (Sn) be S.O.P.S of ∑

and (tn) be S.O.P.S of ∑

as bn ≥ an ∀n∈ℕ

∑ ≥∑

⇒ tn ≥ Sn ∀n∈ℕ ……….(3)

As ∑ = + ∞ ⇒limSn = +∞

(1) ⇒ limtn = + ∞

⇒∑ is divergent

 Notes prepared by sanjeev kumar shukla


 9971245238 or 8750558444
z

Necessary condition of convergence : (n th term test )

If ∑ is convergent then liman = 0

(i.e if liman ≠ 0, ∑ is dgt )

Proof : Let ∑ be cgt, and (Sn) be S.O.P.S then (Sn) is


cgt

⇒limSn = S (say)

Also limSn-1 = S

And an = Sn – Sn-1 ⇒ liman = limSn – limSn-1 = 0

⇒ liman = 0

Geometric test :

A series of the form ∑ for constant a and r is called


Geometric series

∑ is convergent |r|<1

Is divergent if |r|≥1 , a≠0

 Notes prepared by sanjeev kumar shukla


 9971245238 or 8750558444
z

Proof : Let (Sn) be S.O.P.S of ∑

Then Sn = ∑

Now )∑ = ∑ -∑

=a+ar+ar2+……=arn –(ar+ar2+……+arn+1)

=a – ar+n-1

)
⇒∑ = )
r≠1

)
⇒Sn = )

If |r| <1, then lim rn+1 = 0 ⇒ limSn = ( )

⇒S.O.P.S is cgt ⇒ ∑ is convergent

Limit comparision test: Suppose X:= (xn) and Y:=(yn) are


strictly positive sequences and suppose that the following

limit exists in ℝ ; r=lim( )

(a) If r≠0 then ∑ is cgt ⟺ ∑ is cgt


(b) If r=0 then ∑ is cgt ⟺ ∑ is cgt

Proof:

(a) If r≠0 then r>0 ⇒ > 0

And lim = r

∴∃k∈ℕ :
 Notes prepared by sanjeev kumar shukla
 9971245238 or 8750558444
z

| |< ∀n≥k

≤ < r + ∀n≥k

( )< x n < ( 2 r ) y n ∀n≥k ………..(1)

Now yn< xn ∀n≥k

∴Ny comparison test , If ∑ is cgt hen ∑ is cgt

again from (1)

xn< (2r)yn ∀n≥k

⇒ If ∑ is convergent then ∑ is cgt

And hence ∑ is convergent ⟺ ∑ is cgt

(b) If r=0
Then for ε = 1>0 ∃k∈ℕ:

| |< 1 ∀ n ≥ k

⇒ < 1 n≥k

Again by comparision test if ∑ is cgt then ∑ is cgt

 Notes prepared by sanjeev kumar shukla


 9971245238 or 8750558444
z

Root test :

Let ∑ be a sequence and ∝ = limSup ,The series ∑

(i) Converges absolutely if ∝<1

(ii) Diverges if ∝ >1

(iii) otherwise ∝ = 1, and the test giv es no information

Proof:

Case 1 : ∝ >1 ; ∝ = limSup = where we let

Vn = Sup{ : n>N};

As ∝<1, we have 1-∝>0 ; choose 0<ε<1 -e then be


definition of convergence ∃ m ℕ:

|Vn - ∝|<ε ∀N≥m

Or ∝-ε<Vn< ∝+ε ∀N≥m

Or in particular Vn< ∝+ε

⇒ Sup{ : n>N} < ∝+ε

⇒ < ∝+ε ∀n>m

⇒ < (∝+ε)n ∀n>m

As =<∝+ε<1; ⇒∑ ) is convergent

∴By basic comparision test ∑ is abs cgt

 Notes prepared by sanjeev kumar shukla


 9971245238 or 8750558444
z

⇒∑ is convergent

Case 1

∝>1, choose 0<ε<∝-1 then

As ∝ = limSup = ∴ for this ε>0

∃m∈ℕ :

|Vn-∝| < ε ∀ N≥m

⇒∝-ε < Vn ∝+ε ∀N≥m

⇒ Sup{ : n>N} > ∝-ε ∀N≥m

∴ ∃ some N1> N such that

> ∝-ε > 1 ∀N≥m

⇒∃ infinitely many N 1>m such that | > 1

∴ t h e r e i s a s u b s e q u e n c e (| |) o f ) such that each

| |> 1

⇒ lim|an| ≠ 0 ⇒ liman ≠ 0

⇒∑ is divergent (By nth root test)

 Notes prepared by sanjeev kumar shukla


 9971245238 or 8750558444
z

Ratio test : A series ∑ of non-zero terms

(i) Converges absolutely if limSup| |< 1

(ii) Diverges if limInf | |< 1

(iii) Otherwise limInf | | ≤ 1 ≤ limSup| | amd test giver no

information

Proof:

First we’ll show following result

limInf| | ≤limInf ≤limSup ≤limSup| |

The middle inequality is obvious

Now we’ll prove the third inequality

∝ = limSup

And L = limSup| |, w e ’ l l s h o w ∝ ≤ L .

If L = + ∞ then this is obvious (i.e ∝≤L)

If L<+∞. To prove that∝≤L

It is enough to show ∝≤L1 for any L1>L

Since L = limSup| |= | | < L1 ;

There exists a natural number N:

S u p ,| | -< L 1

 Notes prepared by sanjeev kumar shukla


 9971245238 or 8750558444
z

⇒ | |< L 1 f o r n ≥ M

⇒ for n>N, |an| = | || | ……………………. | | |aN|

⇒|an| < |an| for n>N

⇒|an| < a where a = > 0

⇒ limSup ≤ L1 , as desired

⇒∝≤L

⇒ limSup ≤ limSup| | ……………..(1)

Similarly we can show that

limInf| | ≤limInf ………………..(2)

from (1) and (2)

limInf| | ≤limInf ≤limSup ≤limSup| |

If limSup| |< 1

Then limSup < 1

∴ by root test ∑ is cgt

(ii) If limInf| |> 1

⇒ limSup > 1

⇒∑ is divergent by root test

 Notes prepared by sanjeev kumar shukla


 9971245238 or 8750558444
z

(iii) Let ∑ = ∑ then ∑ is dgt

Also limInf| | = limSup| | = lim| | = 1

If ∑ = ∑ then ∑ is dgt

But limInf| | = limSup| | = 1

∴ if limInf| | ≤ 1 ≤ limSup| |

Series ∑ may converge or diverge and hence


we can’t say anyting in this case

Alternative Series :

A series of the form a 1 – a2 + a3 – a4 + ………….. where ak>0 ∀ kj is


called an alternating series

Leibnitz test : If a1≥a2≥a3 …………..

And limak = 0, then the alternating series ∑ ) is cgt

Proof: we’ll prove this using cauchy’s principal of


convergence of an infinite series

Let ε>0 be given

Let ∑ = ∑ )

Consider for n>m , m∈ ℕ

|Um+1 + Um+2 + ……………… + Um|

 Notes prepared by sanjeev kumar shukla


 9971245238 or 8750558444
z

= | ) am+1 + ) am+2 + …....... + ) an|

= | ) [ am+1 am+2 + …....... ) an]|

= | ) A|

Where A = am+1 – am+2 + am+3 - …………….. + ) an

Now a = am+1 – am+2 + ……….. - an

When n-m-1 is odd

A = [ am+1 – am+2 ] + [ am+3 – am+4 ]+ …………….. + [an-1 – an]

{Since (ak) ↓⇒all quantities inside the brackets are +ve


A≥0}

Now

A = am+1 – [am+2 - am+3 ]– [ am+4 –am-5 ]+ ….... + [an-2 – an-1]-an

= am+1 – ve (∵ brackets are non negative )

<am+1 …………….(A)

When n-m-1 is even

A= am+1 – am+2 + am+3 –am+4+ …………….. + an

=[ am+1 – am+2 ]+ [am+3 –am+4 ] + ……….. + [an-2 – an-1] + an

>0 ( ∵ All brackets are non negative )

A = am+1 –[ am+2 + am+3 ] ……….. [an-1 – an-n]

≤am+1 ……………….(B)

 Notes prepared by sanjeev kumar shukla


 9971245238 or 8750558444

You might also like